You are on page 1of 73

AFF SECURITY ANSWERS -BFHLR

ALT/FRAMEWORK

2AC ALT FAILS


Abstract critiques dont actualize into material change
David Chandler 09 - Department of Politics and International Relations, University of
Westminster ("War Without End(s): Grounding the Discourse of `Global War', June 2009,
security dialogue vol. 40 no. 3 243-262, Sage Journals) hk
In focusing on biopower as a means of critiquing universalist policy discourses of global
security, critical theorists of global war from diverse fields such as security studies (Jabri, 2007), development (Duffield, 2007) or critical
legal theory (Douzinas, 2007) are in danger of reducing their critique of war to abstract statements
instrumentalizing war as a technique of global power. These are abstract critiques
because the political stakes are never in question: instrumentality and the desire
for regulation and control are assumed from the outset. In effect, the critical aspect is merely in the
reproduction of the framework of Foucault that liberal discourses can be deconstructed as an exercise of
regulatory power . Without deconstructing the dominant framings of global security
threats, critical theorists are in danger of reproducing Foucaults framework of
biopower as an ahistorical abstraction . Foucault (2007: 1) himself stated that his analysis of biopower was not in any
way a general theory of what power is. It is not a part or even the start of such a theory, merely the study of the effects of liberal governance practices,
which posit as their goal the interests of society the population rather than government. In his recent attempt at a ground-clearing critique of
Foucauldian international relations theorizing, Jan Selby (2007) poses the question of the problem of the translation of Foucault from a domestic to an
international context. He argues that recasting the international sphere in terms of global liberal regimes of regulation is an accidental product of this
move. This fails to appreciate the fact that many critical theorists appear to be drawn to Foucault precisely because drawing on his work enables them
to critique the international order in these terms. Ironically, this Foucauldian critique of global wars has little to do with Foucaults understanding or
concerns, which revolved around extending Marxs critique of the freedoms of liberal modernity. In effect, the post-Foucauldians have a different goal:
they desire to understand and to critique war and military intervention as a product of the regulatory coercive nature of liberalism. This project owes
much to the work of Agamben and his focus on the regulation of bare life, where the concentration camp, the totalitarian state and (by extension)
Guantnamo Bay are held to constitute a moral and political indictment of liberalism (Agamben, 1998: 4).

Understanding the intrincacies of politics, the state, and the military is a


PREREQUISITE to addressing oppression means our ACADEMIC theorizing is
methodologically valuable and a PREREQUISITE to the alternative.
Bryant 12 (9/15, Levi, professor of Philosophy at Collin College and Chair of the Critical
Philosophy program at the New Centre for Research and Practice, War Machines and Military
Logistics: Some Cards on the Table, https://larvalsubjects.wordpress.com/2012/09/15/warmachines-and-military-logistics-some-cards-on-the-table/)

We need answers to these questions to intervene effectively. We can call them questions of military logistics. We are, after all, constructing war
machines to combat these intolerable conditions. Military logistics asks two questions: first, it asks what things the opposing force, the opposing
war machine captured by the state apparatus, relies on in order to deploy its war machine: supply lines,
communications networks, people willing to fight, propaganda or ideology, people believing in the cause, etc. Military
logistics maps all of these things. Second, military logistics asks how to best deploy its own resources in
fighting that state war machine. In what way should we deploy our war machine to defeat war machines like racism, sexism,
capitalism, neoliberalism , etc? What are the things upon which these state based war machines are based, what are the privileged nodes
within these state based war machines that allows them to function? These nodes are the things upon which we want our nomadic war
machines to intervene. If we are to be effective in producing change we better know what the supply lines are so that we might make them our target.

What Ive heard in these discussions is a complete indifference to military logistics . Its as if
people like to wave their hands and say this is horrible and unjust! and believe that
hand waving is a politically efficacious act . Yeah, youre right, it is horrible but saying so doesnt
go very far and changing it. Its also as if people are horrified when anyone discusses anything besides how horribly unjust everything is. Confronted with an
analysis why the social functions in the horrible way, the next response is to say youre
justifying that system and saying its a-okay! This misses the point that the entire
point is to map the supply lines of the opposing war machine so you can
strategically intervene in them to destroy them and create alternative forms of life. You see, we
already took for granted your analysis of how horrible things are. Youre preaching to the
choir . We wanted to get to work determining how to change that and believed for that we needed good maps of the opposing state based war machine so we can decide how to intervene.
We then look at your actual practices and see that

your sole strategy seems to be ideological critique or

debunking . Your idea seems to be that if you just prove that other peoples beliefs are
incoherent, theyll change and things will be different. But weve noticed a couple things about your
strategy: 1) there have been a number of bang-on critiques of state based war machines, without things
changing too much, and 2) weve noticed that we might even persuade others that labor under these ideologies that
their position is incoherent, yet they still adhere to it as if the grounds of their ideology didnt matter much. This leads us to suspect that there are
other causal factors that undergird these social assemblages and cause them to endure is they do. We thought to ourselves, there are two reasons that an ideological
critique can be successful and still fail to produce change: a) the problem can be one of distribution. The critique is
right but fails to reach the people who need to hear it and even if they did receive the message they couldnt
receive it because its expressed in the foreign language of academese which theyve never been substantially exposed to
(academics seem to enjoy only speaking to other academics even as they say their aim is to change the world). Or b) there are other causal factors
involved in why social worlds take the form they do

that are not of the discursive, propositional, or semiotic order. My view is that it

is a combination of both.
I dont deny that ideology is one component of why societies take the form they do and why people tolerate intolerable conditions. I merely
deny that this is the only causal factor. I dont reject your political aims, but merely wonder
how to get there . Meanwhile, you guys behave like a war machine that believes its sufficient to drop pamphlets out of an airplane
debunking the ideological reasons that persuade the opposing forces soldiers to fight this war on behalf of the state
apparatus, forgetting supply lines , that there are other soldiers behind them with guns to their back ,
that they have obligations to their fellows, that they have families to feed or debt to pay off, etc. When I point out these other things its not to
reject your political aims, but to say that perhaps these are also good things to intervene in if we wish to change the world. In other words, Im objecting to your tendency to
use a hammer to solve all problems and to see all things as a nail ( discursive problems ), ignoring the
role that material nonhuman entities play in the form that social assemblages take.
This is the basic idea behind what Ive called terraism. Terraism has three components: 1) Cartography or the mapping of assemblages to understand why they take the form they take and why they endure.
This includes the mapping of both semiotic and material components of social assemblages. 2) Deconstruction Deconstruction is a practice. It includes both traditional modes of discursive deconstruction
(Derridean deconstruction, post-structuralist feminist critique, Foucaultian genealogy, Cultural Marxist critique, etc), but also far more literal deconstruction in the sense of intervening in material or thingly

It is not simply beliefs, signs, and ideologies that cause oppressive social
orders to endure or persist, but also material arrangements upon which people depend to live as
they do. Part of changing a social order thus necessarily involves intervening in
orders upon which social assemblages are reliant.

those material networks to undermine their ability to maintain their relations or feedback mechanisms that allow them to perpetuate certain dependencies for people.
Finally, 3) there is Terraformation. Terraformation is

the hardest thing of all, as it requires the activist to be something

more than a critic , something more than someone who simply denounces how bad things are ,

producing instead other material and semiotic arrangements rendering


new forms of life and social relation possible. Terraformation consists in building alternative forms of life. None of this,
someone more than someone who simply sneers,

however, is possible without good mapping of the terrain so as to know what to


deconstruct and what resources are available for building new worlds . Sure, I care about ontology for political reasons because I
believe this world sucks and is profoundly unjust. But rather than waving my hands and cursing because of how unjust and
horrible it is so as to feel superior to all those about me who dont agree, rather than playing the part of the beautiful soul who refuses to
get his hands dirty, I think we need good maps so we can blow up the right bridges,
power lines, and communications networks , and so we can engage in effective terraformation.
Alt fails cooption political engagement key
McCormack, 10 [Tara, is Lecturer in International Politics at the University of Leicester and
has a PhD in International Relations from the University of Westminster. 2010, (Critique,
Security and Power: The political limits to emancipatory approaches, page 137-138]
In chapter 7 I engaged with the human security framework and some of the problematic implications of emancipatory security
policy frameworks. In this chapter I argued that the

shift away from the pluralist security framework and the


elevation of cosmopolitan and emancipatory goals has served to enforce international power
inequalities rather than lessen them . Weak or unstable states are subjected to greater
international scrutiny and international institutions and other states have greater freedom to intervene, but
the citizens of these states have no way of controlling or influencing these international
institutions or powerful states. This shift away from the pluralist security framework has not
challenged the status quo , which may help to explain why major international institutions and states can
easily adopt a more cosmopolitan rhetoric in their security policies. As we have seen, the shift away from
the pluralist security framework has entailed a shift towards a more openly hierarchical international system, in which states are
differentiated according to, for example, their ability to provide human security for their citizens or their supposed democratic
commitments. In this shift, the old pluralist international norms of (formal) international sovereign equality, non-intervention and
blindness to the content of a state are overturned. Instead, international institutions and states have more freedom to intervene in
weak or unstable states in order to protect and emancipate individuals globally. Critical and emancipatory security theorists

argue that the goal of the emancipation of the individual means that security must be
reconceptualised away from the state. As the domestic sphere is understood to be the sphere of insecurity and
disorder, the international sphere represents greater emancipatory possibilities, as Tickner argues, if security is to
start with the individual, its ties to state sovereignty must be severed (1995: 189). For critical and
emancipatory theorists there must be a shift towards a cosmopolitan legal framework, for example Mary Kaldor (2001: 10), Martin
Shaw (2003: 104) and Andrew Linklater (2005). For

critical theorists, one of the fundamental problems with


Realism is that it is unrealistic. Because it prioritises order and the existing status quo, Realism attempts to
impose a particular security framework onto a complex world , ignoring the myriad threats to
people emerging from their own governments and societies. Moreover, traditional international theory serves
to obscure power relations and omits a study of why the system is as it is: [O]mitting myriad strands of power amounts to
exaggerating the simplicity of the entire political system. Todays conventional portrait of international politics thus too often ends
up looking like a Superman comic strip, whereas it probably should resemble a Jackson Pollock. (Enloe, 2002 [1996]: 189) Yet as I
have argued, contemporary critical security theorists

seem to show a marked lack of engagement with their


problematic (whether the international security context, or the Yugoslav break-up and wars). Without concrete
engagement and analysis, however, the critical project is undermined and critical
theory becomes nothing more than a request that people behave in a nicer way to
each other . Furthermore, whilst contemporary critical security theorists argue that they present a
more realistic image of the world, through exposing power relations , for example, their lack of
concrete analysis of the problematic considered renders them actually unable to engage

with existing power structures and the way in which power is being exercised in the
contemporary international system. For critical and emancipatory theorists the central place of the values of the
theorist mean that it cannot fulfil its promise to critically engage with contemporary power relations and emancipatory possibilities.
Values must be joined with engagement with the material circumstances of the time.

2AC SECURITY RESEARCH GOOD


Debating security is essential to further research skills and to become better
decision makers this is true especially in the context of China
Shaun Breslin 15 - Department of Politics and International Studies, University of Warwick,
Coventry, UK (Debating Human Security in China: Towards Discursive Power?, 2015, Journal
of Contemporary Asia, 45:2, 243-265) hk
hs = human security
Debating and defining HS in China is a task that is closely related to national foreign policy goals and objectives. However, it is
important to remember that it is not just a state project. As we have seen, there are a number of scholars who have a real and deep academic interest in
HS as it is defined outside China, who want to

promote further study and research, and perhaps in the long term
even to influence policy-makers. The states suspicion of any analysis or political action that focusses on the individual means that it
is not always easy to pursue this research agenda. The intellectual space that exists for scholars to debate not just
HS, but issues such as sovereignty and the right to protect can be quickly squeezed out or even
closed off as the political context in which research takes place changes. While there might not be a great deal
of good writing on HS in China, the remarkable thing is that there is as much as there is. In a very modest way, thinking about and responding to HS
concerns and discourses has played a role in changing understandings of the nature of security
in contemporary China. Most clearly, it has contributed to Chinese debates over whether existing
dominant modes of theorising about international relations and security are best placed to
generate effective solutions to new threats. It has also been a factor in rethinking the efficacy of
developing a cooperative foreign policy to deal with new security challenges , and seems to have some
relationship with the evolution of the favoured people-centred harmonious development agenda. The case for influence should not be overstated. If HS
did not exist, these changes might well have emerged in any case. But HS does exist and some scholars have indeed decided to deploy HS concepts with
Chinese characteristics in ways which have fed into other new security discourses to play at least some role in generating change. If HS did not exist, it
is also unlikely that Chinese thinkers would be working to create and define it. But given that it does exist, then it makes no sense to simply let others
define it particularly if they define it in ways that could lead to China being criticised or perhaps even penalised by the international community.

So

by exercising definitional or discursive power, the concept of HS has become Sinicised with a
focus on the most important and imminent HS challenges . As these challenges are primarily rooted in Chinas relative
lack of development, this places human dignity rather than freedom from fear as the fundamental
starting point (Feng 2006; 2010). As a result, promoting socio-economic development is considered the
best way of reducing human insecurity in the Chinese case a task that falls largely to the state (Li 2010). Indeed, the
frequent use of the concept of the security of humanity as a whole rather than the individual human means that the state is central to the discourse of
HS in China in many analyses.

2AC SCENARIO PLANNING GOOD


Scenario analysis is pedagogically valuable enhances creativity and selfreflexivity, deconstructs cognitive biases and flawed ontological assumptions, and
enables the imagination and creation of alternative futures.
Barma et al. 16 (May 2016, [Advance Publication Online on 11/6/15], Naazneen Barma, PhD
in Political Science from UC-Berkeley, Assistant Professor of National Security Affairs at the
Naval Postgraduate School, Brent Durbin, PhD in Political Science from UC-Berkeley, Professor
of Government at Smith College, Eric Lorber, JD from UPenn and PhD in Political Science from
Duke, Gibson, Dunn & Crutcher, Rachel Whitlark, PhD in Political Science from GWU, PostDoctoral Research Fellow with the Project on Managing the Atom and International Security
Program within the Belfer Center for Science and International Affairs at Harvard, Imagine a
World in Which: Using Scenarios in Political Science, International Studies Perspectives 17 (2),
pp. 1-19,
http://www.naazneenbarma.com/uploads/2/9/6/9/29695681/using_scenarios_in_political_s
cience_isp_2015.pdf)
**FYI if anyone is skeptical of Barmas affiliation with the Naval Postgraduate School, its worth looking at her
publication history, which is deeply opposed to US hegemony and the existing liberal world order:
a)

b)

co-authored an article entitled How Globalization Went Bad that has this byline: From terrorism to global
warming, the evils of globalization are more dangerous than ever before. What went wrong? The world
became dependent on a single superpower. Only by correcting this imbalance can the world become a safer
place. (http://cisac.fsi.stanford.edu/publications/how_globalization_went_bad)
most recent published scenario is entitled World Without the West, supports the a Non-Western
reinvention of the liberal order, and concludes that This argument made a lot of people uncomfortable,
mostly because of an endemic and gross overestimation of the reach, depth and attractiveness of the existing
liberal order (http://nationalinterest.org/feature/welcome-the-world-without-the-west-11651)

Over the past decade, the cult of irrelevance in political science scholarship has been lamented by a
growing chorus (Putnam 2003; Nye 2009; Walt 2009). Prominent scholars of international affairs have diagnosed the roots of the gap
between academia and policymaking, made the case for why political science research is valuable for policymaking , and offered
a number of ideas for enhancing the policy relevance of scholarship in international relations and comparative politics (Walt 2005,2011; Mead 2010; Van Evera 2010; Jentleson and Ratner 2011; Gallucci
2012; Avey and Desch 2014). Building on these insights, several initiatives have been formed in the attempt to bridge the gap.2 Many of the specific efforts put
in place by these projects focus on providing scholars with the skills, platforms, and networks to better
communicate the findings and implications of their research to the policymaking community, a necessary and worthwhile
objective for a field in which theoretical debates, methodological training, and publishing norms tend more and more toward the abstract and esoteric.

Yet enhancing communication between scholars and policymakers is only one component of bridging the gap between international affairs theory
and practice. Another crucial component of this bridge is the generation of substantive research programs that
are actually policy relevant a challenge to which less concerted attention has been paid. The dual challenges of bridging the gap are especially acute for graduate
In a field that has an admirable devotion to
pedagogical self-reflection, strikingly little attention is paid to techniques for
students, a particular irony since many enter the discipline with the explicit hope of informing policy.

generating policy-relevant ideas for dissertation and other research topics. Although numerous articles and conference workshops are
devoted to the importance of experiential and problem-based learning, especially through techniques of simulation that emulate policymaking processes (Loggins 2009; Butcher 2012; Glasgow 2012; Rothman
2012; DiCicco 2014), little has been written about the use of such techniques for generating and developing innovative research ideas.

This article outlines an experiential and problem-based approach to developing a political science
research program using scenario analysis. It focuses especially on illuminating the research generation and pedagogical benefits of this technique by describing the use of
scenarios in the annual New Era Foreign Policy Conference (NEFPC), which brings together doctoral students of international and comparative affairs who share a demonstrated interest in policy-relevant
scholarship.3 In the introductory section, the article outlines the practice of scenario analysis and considers the utility of the technique in political science. We argue that scenario analysis should be viewed as a
tool to stimulate problem-based learning for doctoral students and discuss the broader scholarly benefits of using scenarios to help generate research ideas. The second section details the manner in which NEFPC
deploys scenario analysis. The third section reflects upon some of the concrete scholarly benefits that have been realized from the scenario format. The fourth section offers insights on the pedagogical potential
associated with using scenarios in the classroom across levels of study. A brief conclusion reflects on the importance of developing specific techniques to aid those who wish to generate political science scholarship
of relevance to the policy world.
What Are Scenarios and Why Use Them in Political Science?

Scenario analysis is perceived most commonly as a technique for examining the robustness of strategy. It can immerse decision makers in future
states that go beyond conventional extrapolations of current trends , preparing them to take
advantage of unexpected opportunities and to protect themselves from adverse exogenous
shocks. The global petroleum company Shell, a pioneer of the technique, characterizes scenario analysis as the art of considering what if questions about possible future worlds. Scenario
analysis is thus typically seen as serving the purposes of corporate planning or as a
policy tool to be used in combination with simulations of decision making. Yet scenario analysis is not inherently limited
to these uses . This section provides a brief overview of the practice of scenario analysis and the motivations underpinning its uses. It then makes a case for
the utility of the technique for political science scholarship and describes how the scenarios deployed at NEFPC were created.
The Art of Scenario Analysis

We characterize scenario analysis as the art of juxtaposing current trends in unexpected


combinations in order to articulate surprising and yet plausible futures , often referred to
as alternative worlds. Scenarios are thus explicitly not forecasts or projections based
on linear extrapolations of contemporary patterns , and they are not hypothesisbased expert predictions . Nor should they be equated with simulations , which are best
characterized as

functional representations of real institutions or decision-making processes (Asal 2005).

Instead, they are depictions of possible future states of the world , offered together with
a narrative of the driving causal forces and potential exogenous shocks that could lead to those
futures . Good scenarios thus rely on explicit causal propositions that, independent of one another, are plausibleyet, when combined, suggest surprising and sometimes controversial future worlds.
For example, few predicted the dramatic fall in oil prices toward the end of 2014. Yet independent driving forces, such as the shale gas revolution in the United States, Chinas slowing economic growth, and
declining conflict in major Middle Eastern oil producers such as Libya, were all recognized secular trends thatcombined with OPECs decision not to take concerted action as prices began to declinecame
together in an unexpected way.
While scenario analysis played a role in war gaming and strategic planning during the Cold War, the real antecedents of the contemporary practice are found in corporate futures studies of the late 1960s and early
1970s (Raskin et al. 2005). Scenario analysis was essentially initiated at Royal Dutch Shell in 1965, with the realization that the usual forecasting techniques and models were not capturing the rapidly changing
environment in which the company operated (Wack 1985; Schwartz 1991). In particular, it had become evident that straight-line extrapolations of past global trends were inadequate for anticipating the evolving
business environment. Shell-style scenario planning helped break the habit, ingrained in most corporate planning, of assuming that the future will look much like the present (Wilkinson and Kupers 2013, 4).
Using scenario thinking, Shell anticipated the possibility of two Arab-induced oil shocks in the 1970s and hence was able to position itself for major disruptions in the global petroleum sector.
Building on its corporate roots, scenario analysis has become a standard policymaking tool. For example, the Project on Forward Engagement advocates linking systematic foresight, which it defines as the
disciplined analysis of alternative futures, to planning and feedback loops to better equip the United States to meet contemporary governance challenges (Fuerth 2011). Another prominent application of scenario
thinking is found in the National Intelligence Councils series of Global Trends reports, issued every four years to aid policymakers in anticipating and planning for future challenges. These reports present a
handful of alternative worlds approximately twenty years into the future, carefully constructed on the basis of emerging global trends, risks, and opportunities, and intended to stimulate thinking about
geopolitical change and its effects.4 As with corporate scenario analysis, the technique can be used in foreign policymaking for long-range general planning purposes as well as for anticipating and coping with
more narrow and immediate challenges. An example of the latter is the German Marshall Funds EuroFutures project, which uses four scenarios to map the potential consequences of the Euro-area financial crisis
(German Marshall Fund 2013).

Several features make scenario analysis particularly useful for policymaking .5 Long-term global
trends across a number of different realmssocial, technological, environmental, economic, and politicalcombine in oftenunexpected ways to produce unforeseen challenges. Yet the ability of decision makers to imagine,
let alone prepare for, discontinuities in the policy realm is constrained by their existing mental models and
maps. This limitation is exacerbated by well-known cognitive bias tendencies such as
groupthink and confirmation bias (Jervis 1976; Janis 1982; Tetlock 2005). The power of scenarios lies in their
ability to help individuals break out of conventional modes of thinking and analysis by
introducing unusual combinations of trends and deliberate discontinuities in narratives about

the future. Imagining alternative future worlds through a structured analytical


process enables policymakers to envision and thereby adapt to something
altogether different from the known present .
Designing Scenarios for Political Science Inquiry

Scenarios
are essentially textured, plausible, and relevant stories that help us imagine how the future politicaleconomic world could be different from the past in a manner that highlights policy challenges and opportunities. For example, terrorist organizations are a known threat that
The characteristics of scenario analysis that commend its use to policymakers also make it well suited to helping political scientists generate and develop policy-relevant research programs.

have captured the attention of the policy community, yet our responses to them tend to be linear and reactive. Scenarios that explore how seemingly unrelated vectors of changethe rise of a new peer competitor
in the East that diverts strategic attention, volatile commodity prices that empower and disempower various state and nonstate actors in surprising ways, and the destabilizing effects of climate change or
infectious disease pandemicscan be useful for illuminating the nature and limits of the terrorist threat in ways that may be missed by a narrower focus on recognized states and groups. By illuminating the
potential strategic significance of specific and yet poorly understood opportunities and threats, scenario analysis helps to identify crucial gaps in our collective understanding of global politicaleconomic trends and

Very simply, scenario analysis can


throw into sharp relief often-overlooked yet pressing questions in international affairs that
demand focused investigation.
dynamics. The notion of exogeneityso prevalent in social science scholarshipapplies to models of reality, not to reality itself.

Scenarios thus offer, in principle, an innovative tool for developing a political science research agenda . In
practice, achieving this objective requires careful tailoring of the approach . The specific scenario analysis technique we outline below
was designed and refined to provide a structured experiential process for generating problem-based research questions with contemporary international policy relevance.6 The first step in the process of creating
the scenario set described here was to identify important causal forces in contemporary global affairs. Consensus was not the goal; on the contrary, some of these causal statements represented competing theories
about global change (e.g., a resurgence of the nation-state vs. border-evading globalizing forces). A major principle underpinning the transformation of these causal drivers into possible future worlds was to
simplify, then exaggerate them, before fleshing out the emerging story with more details.7 Thus, the contours of the future world were drawn first in the scenario, with details about the possible pathways to that
point filled in second. It is entirely possible, indeed probable, that some of the causal claims that turned into parts of scenarios were exaggerated so much as to be implausible, and that an unavoidable degree of
bias or our own form of groupthink went into construction of the scenarios. One of the great strengths of scenario analysis, however, is that the scenario discussions themselves, as described below, lay bare these
especially implausible claims and systematic biases.8
An explicit methodological approach underlies the written scenarios themselves as well as the analytical process around themthat of case-centered, structured, focused comparison, intended especially to shed

The use of scenarios is similar to counterfactual analysis in that it


modifies certain variables in a given situation in order to analyze the resulting effects (Fearon 1991).
Whereas counterfactuals are traditionally retrospective in nature and explore events that did not actually occur in the context of known history,
our scenarios are deliberately forward-looking and are designed to explore potential
light on new causal mechanisms (George and Bennett 2005).

futures that could unfold. As such, counterfactual analysis is especially well suited to identifying how individual events might expand or shift the funnel of choices available to political actors and thus
lead to different historical outcomes (Nye 2005, 6869), while forward-looking scenario analysis can better illuminate surprising intersections and sociopolitical dynamics without the perceptual constraints

We see scenarios as a complementary resource for exploring these


dynamics in international affairs, rather than as a replacement for counterfactual analysis, historical case studies, or other
imposed by fine-grained historical knowledge.

methodological tools.
In the scenario process developed for NEFPC, three distinct scenarios are employed, acting as cases for analytical comparison. Each scenario, as detailed below, includes a set of explicit driving forces which
represent hypotheses about causal mechanisms worth investigating in evolving international affairs. The scenario analysis process itself employs templates (discussed further below) to serve as a graphical
representation of a structured, focused investigation and thereby as the research tool for conducting case-centered comparative analysis (George and Bennett 2005). In essence, these templates articulate key
observable implications within the alternative worlds of the scenarios and serve as a framework for capturing the data that emerge (King, Keohane, and Verba 1994). Finally, this structured, focused comparison
serves as the basis for the cross-case session emerging from the scenario analysis that leads directly to the articulation of new research agendas.

The scenario process described here has thus been carefully designed to offer some guidance to
policy-oriented graduate students who are otherwise left to the relatively unstructured norms by
which political science dissertation ideas are typically developed. The initial articulation of a dissertation project is generally an idiosyncratic and
personal undertaking (Useem 1997; Rothman 2008), whereby students might choose topics based on their coursework, their own previous policy exposure, or the topics studied by their advisors. Research
agendas are thus typically developed by looking for puzzles in existing research programs (Kuhn 1996). Doctoral students also, understandably, often choose topics that are particularly amenable to garnering
research funding. Conventional grant programs typically base their funding priorities on extrapolations from what has been important in the recent pastleading to, for example, the prevalence of Japan and
Soviet studies in the mid-1980s or terrorism studies in the 2000sin the absence of any alternative method for identifying questions of likely future significance.

The scenario approach to generating research ideas is grounded in the belief that these
traditional approaches can be complemented by identifying questions likely to be of great
empirical importance in the real world, even if these do not appear as puzzles in existing
research programs or as clear extrapolations from past events. The scenarios analyzed at NEFPC envision
alternative worlds that could develop in the medium (five to seven year) term and are designed
to tease out issues scholars and policymakers may encounter in the relatively near
future so that they can begin thinking critically about them now . This timeframe offers a

period distant enough from the present as to avoid falling into current events analysis, but not
so far into the future as to seem like science fiction . In imagining the worlds in which these scenarios might come to pass, participants
learn strategies for avoiding failures of creativity and for overturning the
assumptions that prevent scholars and analysts from anticipating and
understanding the pivotal junctures that arise in international affairs.

High magnitude, low probability scenario planning (like nuclear war) is


productive in IR
Timothy Junio 13, cybersecurity postdoctoral fellow at CISAC, PhD in political science from the
University of Pennsylvania, and Thomas Mahnken, Naval War College, Conceiving of Future
War: The Promise of Scenario Analysis for International Relations, September, International
Studies Review Volume 15, Issue 3, pages 374395
This introduces political scientists to scenarios future counterfactuals and demonstrates their
value
across a wide range of research questions
scenarios contribute to theory building and development , identifying new hypotheses ,
article

in tandem with other methodologies and

. The authors describe best practices regarding the scenario method and argue that

analyzing data-poor research topics , articulating world views , setting new


research agendas , avoiding cognitive biases , and teaching
the low rate at
which scenarios are used in the international relations subfield and situates scenarios in the
broader context of political science methods

social scientists often have a hard time [building scenarios]; they have been
trained to stay away from what if? questions and concentrate on what was?
Scenarioscounterfactual narratives about the
futureare woefully underutilized among political scientists
The low rate at which political scientists use scenarios
is surprising; the method is popular
Scenarios also are a
. The article also establishes

. The conclusion offers two detailed examples of the effective use of scenarios.

In his classic work on scenario analysis, The Art of the Long

View, Peter Schwartz commented that

(Schwartz 1996:31). While Schwartz's comments

were impressionistic based on his years of conducting and teaching scenario analysis, his claim withstands empirical scrutiny.

. The method is almost never taught on graduate student syllabi, and a survey of leading

international relations (IR) journals indicates that scenarios were used in only 302 of 18,764 sampled articles.
than 2% of the time

less

in fields as disparate as business, demographics, ecology, pharmacology, public health, economics, and epidemiology (Venable, Li, Ginter, and

Duncan 1993; Leufkens, Haaijer-Ruskamp, Bakker, and Dukes 1994; Baker, Hulse, Gregory, White, Van Sickle, Berger, Dole, and Schumaker 2004; Sanderson, Scherbov, O'Neill, and Lutz 2004).

common tool employed by the policymakers whom political scientists study. This

article

seeks to

elevate the status of scenarios in political science by demonstrating their usefulness for
theory building and pedagogy . Rather than constitute mere speculation regarding an
unpredictable future

, as critics might suggest,

scenarios assist scholars with developing testable

hypotheses
scenarios are an effective way to teach students to
apply theory to policy
political scientists have invoked the scenario method to improve the specifications of
their theories, propose falsifiable hypotheses, and design new empirical research programs.
What do counterfactual narratives about the future look like?
One of the most common uses
is to study the conditions under which
, gathering data, and identifying a theory's upper and lower bounds. Additionally,

. In the pages below, a best practices guide is offered to advise scholars, practitioners, and students, and an argument is developed in favor of the use of scenarios. The article concludes with two examples

of how

Scenarios

in the Discipline

Scenarios may range in length from a few sentences to many pages.

of the scenario method, which will be referenced throughout this article,

high-consequence , low-probability events may occur. Perhaps the best example of this
is nuclear warfare , a circumstance that has never resulted, but has captivated generations
of political scientists.
weapon might occur

For an introductory illustration, let us

consider a

very simple

scenario regarding how a first use of a nuclear

: During the year 2023, the US military is ordered to launch air and sea patrols of the Taiwan Strait to aid in a crisis. These highly visible patrols disrupt trade off China's coast, and result in skyrocketing

insurance rates for shipping companies. Several days into the contingency, which involves over ten thousand US military personnel, an intelligence estimate concludes that a Chinese conventional strike against US air patrols and naval assets is imminent. The United

States conducts a preemptive strike against anti-air and anti-sea systems on the Chinese mainland. The US strike is far more successful than Chinese military leaders thought possible; a new source of intelligence to the United Statesunknown to Chinese leadership
allowed the US military to severely degrade Chinese targeting and situational awareness capabilities. Many of the weapons that China relied on to dissuade escalatory US military action are now reduced to single-digit-percentage readiness. Estimates for repairs
and replenishments are stated in terms of weeks, and China's confidence in readily available, but dumber, weapons is low due to the dispersion and mobility of US forces. Word of the successful US strike spreads among the Chinese and Taiwanese publics. The
Chinese Government concludes that for the sake of preserving its domestic strength, and to signal resolve to the US and Taiwanese Governments while minimizing further economic disruption, it should escalate dramatically with the use of an extremely small-yield

This
reflects a future event that, while unlikely to occur
contains many dimensions of political science theory . These include
what
leaders perceive as limited, proportional, or escalatory uses of force; the importance of
information about capabilities
the relationship between military
expediency and political objectives during war; and the role of compressed timelines for decision
making
The purpose
is to explain to scholars how such stories, and more rigorously
developed narratives that specify variables of interest and draw on extant data, may improve
nuclear device against a stationary US military asset in the Pacific region.

short story

used for military planning,

and far too vague to be

the following:

private

and commitment; audience costs in international politics;

, among others.

the study of IR .

of this article

An important starting point is to explain how future counterfactuals fit into the methodological canon of the discipline.

2AC SPECIFICITY KEY


Understanding complexities of American foreign policy is crucial to shape the way
we approach the international order in the first place securitization only
happens when we talk about generalities instead of specific mechanisms
Benjamin Valentino 16 - Benjamin is an Associate Professor of Government, Dartmouth
College (Sustainable Security: Rethinking American National Security Strategy, 2016,
http://tobinproject.org/sites/tobinproject.org/files/assets/Sustainable%20Security%20-%20All
%20Chapters.pdf) hk
Understanding the nature of public attitudes about American foreign policy is critical if the
United States wishes to craft a more sustainable national security strategy. Whether America
seeks to achieve sustainability by reining in American commitments or by significantly
reorienting them, in the long run, new policies must be accepted by the American public . Although
it is sometimes asserted that the public plays little role in the realm of foreign policy, the consensus among recent scholarship is

that public opinion has a significant influence over Americas foreign policy
decisions . Contrary to conventional wisdom, attitudes about foreign policy can sometimes be an
important determinant of voting behavior, which means that decision makers who seek to
maintain their positions ignore it at their peril.5 For this reason, every American President since Franklin
Roosevelt has closely tracked public opinion on important foreign policy decisions.6 Because the public is generally not
well informed about foreign affairs, elites probably do have a greater opportunity to shape public
opinion on foreign policy than they do in the domestic arena. Over the longer term, however, if elites cannot convince the
public to support their policies or if the public becomes convinced the policies have failed, they will be punished at the polls.7
Indeed, numerous studies have found that

public opinion can affect important foreign policy

choices, including the timing, duration and conduct of war .8 This chapter, therefore, investigates
American attitudes towards US foreign commitments, drawing on recent public opinion polls including the results of an original
foreign policy survey conducted on a representative sample of Americans in the spring of 2012.9 We know surprisingly little about
US public opinion on these issues. Although pollsters frequently survey American attitudes regarding ongoing military conflicts,

the American public is only rarely asked its opinions on longstanding security commitments to
places like Japan, Korea, Taiwan, Europe, or Kuwaitand even then, often only during times of crisis. Surveys that do
explore foreign policy attitudes tend to address these issues only at the highest level of generality,
such as gauging overall levels of public support for a particular military intervention, public fears about
terrorism, presidential approval in the conduct of foreign policy, American favorability towards specific foreign
countries, and attitudes about the defense budget. To understand how the public is likely to react to proposals
for significant changes in American grand strategy, however, we need to know not simply what
the public thinks, but why

2AC REFORMISM GOOD


Single issue focus is comparatively better than ideological critique the their
pedagogy gets sidelined and perpetuates the status quo flips all of their offence
about systemic critique and disproves the veracity of all of their framework and
alternative arguments
Lieberfeld, Associate Professor of Public Policy @ McAnulty College, 8
(Daniel, WHAT MAKES AN EFFECTIVE ANTIWAR MOVEMENT? THEME-ISSUE
INTRODUCTION, International Journal of Peace Studies, Volume 13, Number 1,
https://www.gmu.edu/programs/icar/ijps/vol13_1/IJPS13n1%20Intro%20-%20Lieberfeld.pdf)
Important definitional considerations concern the temporal and issue dimensions of antiwar
protest. Opposition to a particular war motivates some movements. These ad hoc movements
seek to change government policy regarding a specific, ongoing war. While this goal may
be linked to other political agendasbe they anti-militarist , feminist, anti-imperialist ,
pro-democracy, and so forththese are secondary to the primary focus on bringing a particular
war to an end. The time-horizon of such movements is limited and they typically dissolve or
become inactive after the war ends. A different type of antiwar activism transcends protest
against specific wars. It has a more extensive temporal dimension and greater prominence
of ideologically based motives and goals such as pacifism, or liberal internationalism
that seeks to institutionalize world order through the United Nations or a federation of
countries. Ongoing protests by secular pacifist groups or by peace churches against armaments
and militarism can have much more diffuse goals than do ad hoc antiwar movements. In
addition to disarmament, these goals may include strengthening of international
disputeresolution processes, promoting international understanding, and peace education. One
can designate ad hoc protests antiwar movements and more ideologically motivated and
long-running protests peace movementsalthough these categories are not mutually exclusive
and protesters against particular wars may also have transcendent ideological motives. Openended, more ideologically motivated movements may have less potential , at least in the
near term, to influence public opinion and change public policy . In part this reflects the
more diffuse goals of ongoing peace movements: Insofar as a single-issue focus tends to
correlate with greater ability to achieve movement goals (Gamson, 1990, 45-46), ad hoc antiwar
groups may be more successful. Of course, the task of ending a particular war is more
achievable than that of ending war generally . Peace groups whose demands include the
expansion of international law at the expense of state sovereignty are also politically radical in
the sense that they challenge present distributions of wealth and power, and advocate
replacing the authority over security policy claimed by domestic elites (Ash, 1972, 230). Scholars
debate how the radicalism of a movements demands affects its prospects for success, but the
goal of displacing established political authorities is highly correlated with protest-group failure
(Gamson, 1990, 42). Antiwar groups that focus on ending a particular war do not generally seek
to replace the authority of domestic elites. Rather, their challenge is directed toward
particular policies and practices that they believe depart from the responsible exercise of

officeholders authority. Thus, a key question that bears on questions of antiwar movements
effectiveness and influence is What is the relative importance t
o movement leaders of ideological goals broader than ending a particular war? It may also be
useful to locate movement goals on a continuum from domestic to international politics, with
world peace and disarmament goals located on the more international and abstract end and also
implying the potentially radical displacement of domestic elites. Peace groups with broader
time horizons and more abstract goals generally find it harder to achieve favorable
public responses and typically remain politically marginal. Ad hoc protests with a single
agenda of ending a war have greater potential to attract mainstream support and to
contribute to changes in policy.

Reformism is effective
Richard Delgado 9, self-appointed Minority scholar, Chair of Law at the University of
Alabama Law School, J.D. from the University of California, Berkeley, his books have won eight
national book prizes, including six Gustavus Myers awards for outstanding book on human
rights in North America, the American Library Associations Outstanding Academic Book, and a
Pulitzer Prize nomination. Professor Delgados teaching and writing focus on race, the legal
profession, and social change, 2009, Does Critical Legal Studies Have What Minorities Want,
Arguing about Law, p. 588-590
CLS critique of piecemeal reform Critical scholars reject the idea of piecemeal reform.
Incremental change, they argue, merely postpones the wholesale reformation that must occur to
create a decent society. Even worse, an unfair social system survives by using piecemeal reform to
disguise and legitimize oppression. Those who control the system weaken resistance by pointing
to the occasional concession to, or periodic court victory of, a black plaintiff or worker as evidence that
the system is fair and just. In fact, Crits believe that teaching the common law or using the case method in law school is a disguised means of preaching
2. The

incrementalism and thereby maintaining the current power structure. To avoid this, CLS scholars urge law professors to abandon the case method, give up the effort to nd
rationality and order in the case law, and teach in an unabashedly political fashion.

The

CLS

critique of piecemeal reform is

familiar, imperialistic and wrong . Minorities know from bitter experience that occasional court victories do not mean the Promised Land
is at hand. The critique is imperialistic in that it tells minorities and other oppressed peoples how they should
interpret events affecting them. A court order directing a housing authority to disburse funds
for heating in subsidized housing may postpone the revolution, or it may not . In the
meantime, the order keeps a number of poor families warm . This may mean more to them
than it does to a comfortable academic working in a warm office. It smacks of paternalism to
assert that the possibility of revolution later outweighs the certainty of heat now , unless there is
evidence for that possibility. The

Crits do not offer such evidence. Indeed, some incremental changes may bring

revolutionary changes closer , not push them further away. Not all small reforms induce
complacency ; some may whet the appetite for further combat. The welfare family may hold a
tenants union meeting in their heated living room. CLS scholars critique of piecemeal reform
often misses these possibilities, and neglects the question of whether total change, when it
comes, will be what we want.

Single issue rights focus spills over and is distinct from the ideologies that
determine its use
Hunt 90 (Alan, Professor of Law and Sociology, Carleton University, Ottawa, Canada, Rights
and Social Movements: Counter-Hegemon Strategies, Journal of Law and Society Vol. 17 No. 3,
1990)

Beyond questions concerning the criteria of 'success' there is another and perhaps more
fundamental problem with the existing studies of the use of litigation by social movements.
There is a failure to distinguish between the very different types of social movements that have
been studied.26 What is missing is a concern with what I propose to call the 'hegemonic
capacity' of social movements. In a first approximation the distinction can be drawn between
'single issue' movements and those whose goals would constitute a wider set of social
changes than their immediate objectives. But this approximation requires further refinement
because some movements which are apparently single issue have extensive ramifications .
The abortion rights movement, whilst superficially focusing on a single issue, has ramifications
extending beyond the immediate question of women's right to control their fertility. The
abortion rights movement is a prime example of the concept of 'local hegemony'. Such a
movement is not directed to the kind of global hegemony that Gramsci had in mind with his
focus on the role of the revolutionary party. But movements directed towards local (or regional)
hegemony can only be adequately judged in their capacity to transform a wide range of
social practices and discourses. For present purposes I suggest that, in addition, the
environmental movement and the civil rights movement also serve as my example of
movements of 'local hegemony' in that while focused on a set of specific demands , their
realization would both necessitate and occasion wider structural changes . The most
immediate implication is that their 'success' is not a matter of securing some immediate interest.
It follows that to evaluate the role of litigation for such movements necessitates that focus be
directed to the articulation between the elements that make up the strategic project of the
movement. My suggestion is that a key feature of any such assessment revolves around their
capacity to put in place a new or transformed discourse of rights which goes to the heart of the
way in which the substantive issues are conceived, expressed, argued about, and struggled over.
My more controversial suggestion is that the immediate 'success' or 'failure' of specific litigation
has to be approached in a different way which requires that we take account of the possibility
that litigation 'failure' may, paradoxically, provide the conditions of 'success' that compel a
movement forward. In current struggles over wife abuse, all those cases in which judges impose
derisory sanctions are contexts which drive the movement forward because they provide

instances of a dying discourse in which women 'deserve' chastisement by their husbands. Such
judicial pronouncements become more self-evidently anachronistic and in this inverted form
speak of a new and emergent discourse of rights and autonomy. The implications of this line of
thought are that the whole question of the success or failure of litigation and its connection with
transformative strategies is far more complex than our existing attempts to measure 'success'
and 'failure' admit.
A more far-reaching criticism of litigation is that, rather than helping, 'law', conceived variously
as litigation or legal reform politics, is itself part of the problem. This line of argument is at the
root of Kristin Bumiller's study of the civil rights movement.27 This strand of the anti-rights
critique is, I want to suggest, even if unintended, a form of 'Leftism' whose inescapable error
lies in the fact that it imagines a terrain of struggle in which a social movement can, by an act of
will, step outside the terrain on which the struggle is constituted, Here a hegemonic
strategy must insist that it is precisely in the engagement with the actually existing
terrain , in particular, with its discursive forms, that the possibility of their
transformation and transcendence becomes possible. To refuse this terrain is, in
general, Leftist because is marks a refusal to engage with the conditions within which social
change is grounded.

The neg has failed to engage the institutions that exist and control society -- We
must move beyond their abstraction in favor of discussing the tough choices and
trade-offs that a non-institutional analysis wishes away institutions are
inevitable and learning to pragmatically engage them best facilitates change
empirically proven through New York State prison reform that effectively included
the work of prisoners
Minkler 12 [Meredith Minkler, Professor of Health and Social Behavior, University of
California-Berkeley, Community Organizing and Community Building for Health and Welfare,
New Brunswick: Rutgers, 2012]

In public health, urban and regional planning, social work, and related fields, a hallmark of community
organizing and community building lies in their commitment to action and social change . Although
such action may take many forms, community builders and organizers increasingly are turning to policy
approaches as among the most potent for affecting the health and well-being of communities .
The rationale for an emphasis on policy in the health field is well documented . The dramatic
declines in U.S. mortality rates over the past century have been attributed in large part to
environmental and policy-related changes in sanitation, water supply, and food quality (McGinnis
and Foege 1993; House et al. 2008). More recently, community organizing and the subsequent development of social
movements in areas such as women's rights , HIV/AIDS prevention , disability rights ,

and environmental and climate justice have played a crucial role in changing policies on the local
through the national levels (Brown et al. 2012). Successful efforts in many parts of the country to ban
smoking in public places, curb the sale of handguns, and promote healthier food environments,
as well as the enactment of legislation (albeit inadequate) to clean up toxic waste, are among the many policyrelated victories that took root in local community organizing and community
building efforts. Finally, on a global level, the healthy cities/healthy communities movement has, since its inception,
focused on broad policy-level changes as a means of helping communities realize their visions of a healthy place in which to live,
work, and play (Corburn 2009; O'Neill and Simard 2006; see chapter 9). For

many community residents, however,


"public policy has become unfamiliar and irrelevant, complicated, inaccessible and confusing "
(Blackwell and Colmenar 2000, 162). And even those who do believe in the importance of public policy
often feel ineffectual in their ability to influence major decisions affecting their lives . In a similar
way, professionals in health, social work, and related fields who work on the community level
sometimes have been reluctant to focus on policy-related activity, which they perceive as taking place
primarily "out there" on the state and national levelslevels seen as being far removed from their day-to-day community organizing
efforts (Minkler and Freudenberg 2010). Confusion over the extent to which nonprofit organizations can engage in policy advocacy
without jeopardizing their funding also have been cause for concern, though as Homan (2011) and others have noted, this fear is
often exaggerated and not in line with actual federal or foundation grantee guidelines (see chapter 7). Despite

the

challenges faced, health professionals and their community partners, as well as policymakers themselves, increasingly
are recognizing community organizing and community building as critical strategies for
helping to effect healthy public policy . Indeed, if community building principles are taken
seriously, policymaking itself may become a process of community building , with
community members engaged at every step, from framing the issues to interpreting the data,
discussing the options, and working for the adoption of the policy change they wish to
see. We begin this chapter by offering a conceptual framework for understanding policy and policy advocacy from a community
organizing and community building perspective, including a look at the key steps involved in this process. We then offer two
examples demonstrating some of the many ways in which local community organizing and community-based partnership efforts
have worked to influence the policymaking process. We conclude by broadening our gaze to suggest how the application of
community building and community organizing principles should be an integral part of policy design. Conceptual Framework For
the purposes of this chapter, policies

are defined as the laws and regulations, including both formal and
informal rules, "by which opportunities are framed what is allowed, encouraged, discouraged, or prohibited" (Bell and
Standish 2005, 339). As Bell and Standish go on to note, " Policies also determine the shape, size and character
of communities," including, for example, neighborhood density and population composition, and whether businesses
(including polluting industries) can move in (339). As Milio (1998) has pointed out, "The intent [of policy] is to achieve a more
acceptable state of affairs and, from a public health perspective, a more health- promoting society" (15). From the perspective of
health equity, policies should be designed that "[provide] all people with fair opportunities to attain their full health potential to the
extent possible" (Braveman 2006, 167), in large part by working to eliminate unfair or unjust conditions resulting in health
disparities (see also Giles and Liburd 2010; Bell and Lee 2011). Many

models of the policymaking process have


relevance for those engaged in community organizing and community building . Among the most
influential is a model developed by political scientist John Kingdon (2003) that posits that in order to get the
attention of policymakers, we need to address three processes : convincing decision makers
that a problem exists ; proposing feasible, politically attractive proposals to solve
the problem ; and negotiating the politics that influence whether a proposal
succeeds in the political arena . The first stage, moving a policy issue onto the political agenda, is often the starting
point for community organizing, and Kingdon's model has been used to help examine community- based agenda setting on issues
such as teen access to birth control (Brindis et al. 2009), food security (Brechwich Vasquez et al. 2007), and asthma prevention
through environmental policy change (Minkler et al. 2010). Theorists

also stress the importance of a "window of


opportunity" for policy that opens when there is a favorable coming together of problems,

potential solutions, and political circumstances, often accompanied by community or public


engagement (Kingdon 2003). Identifying opening "policy windows" and being able to jump through
them are important skills for community organizers. Monitoring developments in all three
processes (getting your problem on the agenda, proposing a feasible solution , and getting
to passage and implementation) is also a critical task for organizers and their allies, who can be
ready to move in any of the three arenas as needed (Minkler and Freudenberg 2010). Health policy researcher
Beauford B. B. Longest Jr. (2006) describes the policy process as highly political, reflecting a mix of influences on both public and
individual interests and rarely proceeding on the basis of rational or empirical decision making. Core components of his model
suggest that it is also "distinctly cyclical," with a circular flow of interactions and influences between the various stages, and that it
involves "an open system" in which "the process interacts with and is affected by events and circumstances in its external
environment." The three distinct but interconnected phases in this model are the following: Policy formulation, encompassing
activities involved in setting the policy agenda and later in the actual development of legislation Policy implementation, involving
all those activities connected with rule making to guide the actual implementing and operationalizing of a policy Policy
modification, including the revisiting and possible alteration of all previous decisions and compromises made during the
policymaking process As Longest's model suggests, the

formal enactment of legislation serves as a bridge


between the policy formulation process and the subsequent implementation phase . The policy
modification phase then comes into play as a feedback loop through which minor tweaking of the legislationor a major revisiting of
the agenda-setting processmay take place. Both

the political nature of policymaking and the dynamic


nature of the external environment in which policymaking occurs underscore the likelihood of
policy modificationand in extreme cases, even repealduring this phase of the process. Finally, both these and
other popular models of the policymaking process (Milio 1998; Bardach 2000) suggest the existence of a series of steps or stages
through which such processes typically progress, albeit with many feedback loops and circuitous detours along the way. Drawing on
these various models, and on the work of organizer Makani Themba-Nixon (1999, 2010), we summarize these stages and
corresponding roles for organizers and advocates along the way: Problem identification and refinement. Policymakers and
advocates identify problems or issues to be addressed in future legislation. Advocates and community organizers may "test the
waters" at this stage, working with community members and conducting research to identify a shared problem or vision and to
discover the true extent of support (or opposition) it may entail. As Themba-Nixon (2010) points out, "A good organizer carefully
dissects how key constituents will perceive their self interests as they relate to the proposed policy" (147). Setting an agenda and
creating awareness. In policy terms, as Kingdon (2003) notes, the agenda is the list of issue areas to which policymakers are paying
attention. Town hall meetings, public and policy awareness campaigns, media advocacy, and other means (e.g., testifying at hearing)
all may be critical in both creating wider community awareness and getting on the policy agenda. Setting policy objectives. Clear
objectives for the policy are identified based on the problem-reframing process. A SWOT analysis may be conducted at this stage to
help community members lay out the strengths and weaknesses internal to their case and the opportunities and threats in the larger
environment (Barry 1997; see also appendix 7). Designing alternative courses of action and weighing their consequences. In this
"marketplace stage" competing interests are offered and compromises discussed. Advocates, organizers, and community members
assess the likelihood of winning on their initial proposal and discuss alternatives if these seem necessary. Having a clear "pocketbook
angle" (Staples 2004; see chapter 11) may also be critical at this stage to be able to show that the advocates' proposed solution is
ideally cost saving, or at minimum cost neutral or worthy of the price tag attached. Victory and defense. Although not typically
listed in policy process models, community celebrations of victories and attaining media exposure for these wins is an important
part of the process from a community organizing and community building perspective. Concurrent with this stage, however, legal
and other challenges must be anticipated and prepared for (Themba 1999; Temba-Nixon 2010). Assigning implementation
responsibility. Decision makers identify the agency or unit with responsibility (and ideally the resources) for implementation;
community groups work to ensure that enforcement takes place and that oversight mechanisms are in place. Evaluation. Many
methods are employed for assessing policy impact and outcomes, such as stakeholder interviews, case studies, and tracking of media
coverage, as well as more tailored advocacy evaluation methods, including policymaker ratings and the Bellwether methodology,
through which a sample of influential and knowledgeable people in the public or private sectors are interviewed concerning a range
of current policy issues (Coffman 2002; Coffman and Reed 2009). From a community organizing and community building
perspective, participatory or empowerment evaluation (Wallerstein et al. 2002; Fetterman and Wandersman 2005) ideally is
employed throughout the process to enable community members to play an active role in the evaluation procedure (see chapter 19).
As suggested above, and despite

the linear listing of stages in policy process models like that described
here, this process is in reality a dynamic and frequently convoluted one. The often substantial
role of private interest groups and political considerations also should not be underestimated, as
captured in the saying popularized by Otto van Bismarck over a century ago, that "laws are like sausages. You should never watch
them being made." Yet despite the messiness, involvement in the policymaking process has never been more vital for those
concerned with community building and improving the public's health. We turn now to two brief case studies illustrating successful
efforts by diverse grassroots groups, community-based organizations, and coalitions, often aided by professionals in public health

and related fields, to create and shape healthy public policy. Influencing Policy through Community Organizing and Community
Building: Case Examples Changing Policies to Foster Reintegration of Inmates Leaving Jail in New York City In

New York
City, where nearly one hundred thousand inmates are released from prison or jail annually,
many return to jail or prison within a year, making their release, not one way, but a "round trip."
(Freudenberg et al. 2005). The communities of Central and East Harlem, whose returning inmates often are incarcerated for crimes
related to poverty, violence, substance abuse, and mental illness, have been particularly affected by this confluence of problems
(Freudenberg 2001; Freudenberg et al. 2005; Van Olphen et al. 2003; Freudenberg et al. 2010). The

intersection of
reentry issues with substance abuse, the longtime focus of the Center for Urban Epidemiologic
Studies (CUES) in New York, led its Policy Work Group to identify reintegration of drug users
who have served time in jail and prison as a priority concern.

CUES included as partners

community residents and representatives of advocacy organizations, the New York City Department of Health and Mental Hygiene,
and academics from Hunter College and the New York Academy of Medicine. An active Community Advisory Board also worked
with CUES throughout the process. Together, the diverse membership of the Policy Work Group recast the problem of inmate
reentry in a public health framework. They

noted, for example, that high proportions of returning inmates had


substance abuse and other mental health problems, and frequently conditions such as
HIV/AIDS, tuberculosis, and hepatitis C as well. They further defined the problem in terms of such barriers to
reintegration as the fact that 90 percent of returning inmates had not completed high school and many were impoverished,
homeless, and unemployed (Freudenberg 2001; Freudenberg et al. 2005, 2010; Van Olphen et al. 2003). As noted above, critical to
translating a community issue into an actionable policy agenda is "doing your homework," and finding out the extent of the problem
and its ramifications. The Policy Work Group undertook a wide range of fact-finding methods, including secondary data analysis
regarding substance abuse and incarceration, focus groups with former inmates, survey research with providers, public opinion
polling, and policy analysis. Together,

this research helped uncover eleven different policies that

appeared to work against successful community reintegration. For example, the policy of
terminating, rather then temporarily suspending, Medicaid coverage for inmates while they were incarcerated often precluded their
getting timely access to substance abuse counseling and treatment on their release, contributing, for many, to the "round trip"
nature of their incarceration (Freudenberg et al. 2005; Van Olphen et al. 2003). To

increase awareness of the


problem among community members and policymakers, the Policy Work Group shared its
findings through community forums and facilitated discussions that in turn helped further hone
their policy objectives and targets. Consistent with Themba's (1999; Themba-Nixon 2010) reminder to begin by
identifying the individuals or institutions with the power to address the community's demands and help ameliorate the problem,
CUES, its Community Advisory Board, and its Policy

Work Group identified and targeted those

government entities with the power to make the relevant policy changes they sought

among them New York's Department of Correction, the city council, the mayor,

and the state legislature . Further, and in keeping with Staples' (2004; chapter 11) reminder to address the
"pocketbook angle" in desired policy changes, the Policy Work Group advocated for and got an Independent Budget Office study of
the annual costs of reincarceration, using the results to further push for policy change. A key finding of this analysisthat having
Medicaid in the year after release significantly reduced reincarceration rates by two-thirds (Lee et al. 2006) packed substantial
"political punch" (chapter 9) demonstrating cost savings to the city and state and helping strengthen the case for policy change.

CUES and its Policy Work Group also played a key role in helping bring together a broader
citywide coalition, the Community Reintegration Network, which also pushed for action from the New York
City Council and the mayor. Their policy reports, among them Coming Home from Jail: An Action Plan to Improve NYC Reentry
Policies and Programs (Hunter College and Community Reintegration Network 2003) were widely distributed to city officials and
other stakeholders and included both the human and fiscal costs of current policies and alternative approaches. Throughout

this process, moreover, community members affected by the problem substance abusers
who were former inmates continued to play a key role. In the words of one such
participant: "All the work done in the Policy Group has been informed by the
action people who have come out of the prisons and jails.

I think that has kind of grounded our

work and also gives strength to the work we do. Because it isn't just us thinking about it from the policy point of view, but actually
from the affected folks that are saying 'these are the problems we are facing.'" CUES

and its Policy Work Group's

leadership and organization, together with the effective advocacy of the Community
Reintegration Network, contributed to a number of important changes in programs, practices
and policies. Among these were the Department of Correction's agreement, in 2004, to release most inmates during
daylight hours rather than in the middle of the night, and city legislation that same year
mandating discharge planning and relevant social services for all inmates prior to release. The
latter policy was expanded the following year to include help finding housing and employment.
The Community Reintegration Network and other institutions also worked to create new or expanded programs for reentering
inmates, including job training, follow-up services, and health care. Finally, and of perhaps greatest importance, the

Policy
Work Group and its allies were described by policymakers and others as having played a key role
in getting the New York State legislature to pass a bill in 2007 reinstating Medicaid
coverage to inmates upon their release, replacing the previous policy that had
permanently revoked benefits upon incarceration .

AT: FIAT IS VIOLENT


Responding to the question what should we do? is unavoidable because it is
posed by the debate round itself how we research, how we prepare, what
arguments we choose to read, what questions we make the debate normative
ethical questions are inevitable, its only a question of which questions we choose.
Our argument is that we are morally obligated to direct our normative ethical
questions towards cultivating informed compassion through an understanding of
political action. This is uniquely true in the pedagogical space of the debate round
Ruiz and Minguez 1 [Prof. Dr Pedro Ortega Ruiz, Facultad de Educacio n, Campus de
Espinardo, Universidad de Murcia, Global Inequality and the Need for Compassion: issues in
moral and political education Journal of Moral Education, Vol. 30, No. 2, 2001]

What can pedagogy do? We believe that its intervention must be founded upon the training of a
critical conscience, liberating oppressor and oppressed alike and in the recovery of lost dignity
in human relationships. That is to say, to contribute to the facilitation of a new state of things, a new
society based upon justice and equality. This implies overcoming first the reductionist
conception of education centred around the acquisition of knowledge which has formed the
backbone of the school curriculum and to place the learning of values at the heart of the
educational process. It supposes starting from the context or reality in which those to be educated live so that the context is
the necessary starting point of the educational process and the reality is the arrival point so that it is possible to know it, form part of

Educational processes are not found in a no mans land but rather in the
immediacy of shared meanings in which it is possible to establish human communication. Man
is not educated in the abstract but in the particular and concrete, bound in a similarly concrete
reality. In pedagogy it is customary to find a tendency of flight from reality, especially when this
is disturbing or unpleasant. Frequently pedagogic discourse hides behind idealistic formulae
that are none other than an inability to face real situations. In pedagogy we cannot avoid the
question, What should be done? because the practical component is at the very centre of the
teaching debate. However, the question is never posed without reference to concrete situations, to
provide answers to specific problems. Being true to reality forces us to ask about what is going
on, to criticise the conditions which form the situations of life of these subjects if what we seek is
education of the whole person. One without the other leads to alienation, an intellectual exercise
as interesting as it is sterile. Therefore a Copernican shift is required on the key points of our educational action, still
it and change it.

anchored in an academic conception, to base it upon the development and learning of attitudes and values, skills and abilities for a
critical and creative learning, for co-operation and participation, to become part of reality, judge and transform it. It requires
thinking of and carrying out education in a different way, to learn for life. It also requires the commitment to an education for
compassion, for the assumption of the others cause, that of the defenceless and oppressed. That is to say, it requires a style of
education which perceives the awareness of anothers suffering as an injustice, accepting our own responsibility towards the other
and our responsibility for the restoration of his/her dignity. We speak therefore of an empathic understanding which places the
individual in the face of situations of exclusion and misery not as a simple spectator, but as someone on whom also depends the
removal of such situations of indignity and suffering.

AT: OBJECTIVITY BAD


Valid, descriptive theories of the world are an essential prerequisite to
emancipatory critique
Jones 04 (August 2004, Branwen Gruffydd, PhD in Development Studies from the
University of Sussex, Senior Lecturer in International Political Economy at Goldsmiths
University of London, From Eurocentrism to Epistemological Internationalism: power,
knowledge and objectivity in International Relations, Paper presented at Theorising Ontology,
Annual Conference of the International Association for Critical Realism, University of
Cambridge, http://www.csog.group.cam.ac.uk/iacr/papers/Jones.pdf)

The common-sense view pervading recent discussions of epistemology, ontology and methodology in IR asserts that objectivity implies
value-free neutrality. However, objective social inquiry has an inherent tendency to be
critical , in various senses. To the extent that objective knowledge provides a better and more
adequate account of reality than other ideas, such knowledge is inherently critical
(implicitly or explicitly) of those ideas . 30 In other words critical social inquiry does not (or
not only) manifest its criticalness through self-claimed labels of being critical or
siding with the oppressed, but through the substantive critique of prevailing ideas .
Objective social knowledge constitutes a specific form of criticism: explanatory critique . The
critique of dominant ideas or ideologies is elaborated through providing a more adequate explanation
of aspects of the world, and in so doing exposing what is wrong with the dominant
ideology . This may also entail revealing the social conditions which give rise to ideologies, thus exposing the necessary and causal relation between particular social relations and particular
ideological conceptions.

the reproduction of those structured


relations is in the interests of the powerful, whereas transformation of existing structured relations is in the
interests of the weak. Because ideas inform social action they are casually efficacious
either in securing the reproduction of existing social relations (usually as an unintended consequence of social practice), or in informing social
action aimed at transforming social relations . This is why ideas cannot be neutral . Ideas which provide a misrepresentation of the nature
In societies which are constituted by unequal structures of social relations giving rise to unequal power and conflicting interests,

of society, the causes of unequal social conditions, and the conflicting interests of the weak and powerful, will tend to help secure the reproduction of prevailing social relations. Ideas which provide a more
adequate account of the way society is structured and how structured social relations produce concrete conditions of inequality and exploitation can potentially inform efforts to change those social relations. In

ideas which are false are ideological and, in serving to promote the reproduction of the status quo and
avoid attempts at radical change, are in the interests of the powerful. An account which is objective will contradict ideological ideas, implicitly or
explicitly criticising them for their false or flawed accounts of reality. The criticism here arises not, or not only, from pointing out the coincidence between ideologies and the
interests of the powerful, nor from a prior normative stance of solidarity with the oppressed , but from exposing
this sense,

the flaws in dominant ideologies through a more adequate account of the nature and
causes of social conditions 31 .
A normative commitment to the oppressed must entail a commitment to truth and
objectivity, because true ideas are in the interest of the oppressed, false ideas are
in the interest of the oppressors . In other words, the best way to declare solidarity with the

oppressed is to declare ones commitment to objective inquiry 32 . As Nzongola-Ntalaja


(1986: 10) has put it:
It is a question of whether one analyses society from the standpoint of the dominant groups ,
who have a vested interest in mystifying the way society works, or from the standpoint of
ordinary people, who have nothing to lose from truthful analyses of their predicament .
The philosophical realist theory of science , objectivity and explanatory critique thus
provides an alternative response to the relationship between knowledge and power .
Instead of choosing perspectives on the basis of our ethical commitment to the
cause of the oppressed and to emancipatory social change, we should choose between
contending ideas on the basis of which provides a better account of objective
social reality . This will inherently provide a critique of the ideologies which, by virtue of their flawed account of the
serve the interests of the powerful.

social world,

Exemplars of explanatory critique in International Relations are provided in the work of scholars such as Siba Grovogui, James Gathii, Anthony Anghie, Bhupinder Chimni, Jacques Depelchin, Hilbourne Watson,
Robert Vitalis, Sankaran Krishna, Michel-Rolph Trouillot 33 . Their work provides critiques of central categories, theories and discourses in the theory and practice of IR and narratives of world history, including
assumptions about sovereignty, international society, international law, global governance, the nature of the state. They expose the ideological and racialised nature of central aspects of IR through a critical
examination of both the long historical trajectory of imperial ideologies regarding colonized peoples, and the actual practices of colonialism and decolonisation in the constitution of international orders and local
social conditions. Their work identifies the flaws in current ideas by revealing how they systematically misrepresent or ignore the actual history of social change in Africa, the Caribbean and other regions of the
Third World, both past and present during both colonial and neo-colonial periods of the imperial world order. Their work reveals how racism, violence, exploitation and dispossession, colonialism and neocolonialism have been central to the making of contemporary international order and contemporary doctrines of international law, sovereignty and rights, and how such themes are glaring in their absence from
histories and theories of international relations and international history.

Objective social knowledge which accurately depicts and explains social reality has these
qualities by virtue of its relation to its object, not its subject . As Collier argues, The science/ideology distinction is an
epistemological one, not a social one. (Collier 1979: 60). So, for example, in the work of Grovogui, Gathii and Depelchin, the
general perspective and knowledge of conditions in and the history of Africa might be due largely to the African social
origins of the authors. However the judgement that their accounts are superior to those of mainstream IR rests
not on the fact that the authors are African, but on the greater adequacy of their accounts with
respect to the actual historical and contemporary production of conditions and change in Africa and
elsewhere in the Third World. The criteria for choosing their accounts over others derives from the relation between the ideas and their objects (what they are about), not from the relation between the
ideas and their subjects (who produced them). It is vital to retain explicitly some commitment to objectivity in social inquiry, to the notion that the proper criterion for judging ideas about the world lies in what
they say about the world, not whose ideas they are.

A fundamental problem which underlies the origin and reproduction of IRs eurocentricity is the overwhelming
dominance of ideas produced in and by the west, and the wilful and determined silencing of the
voices and histories of the colonised . But the result of this fundamental problem is
flawed knowledge about the world . Eurocentricity is therefore a dual problem
concerning both the authors and the content of knowledge, and cannot be
resolved through normative commitments alone . It is not only the voices of the colonised, but the histories of colonialism, which have
been glaring in their absence from the discipline of International Relations.

Overcoming eurocentricity therefore requires not only concerted effort from the centre to create
space and listen to hitherto marginalised voices, but also commitment to correcting the
flaws in prevailing knowledge and it is not only the Other who can and should
elaborate this critique . A vitally important implication of objectivity is that it is the responsibility of European and
American, just as much as non-American or non-European scholars, to decolonise
IR . The importance of objectivity in social inquiry defended here can perhaps be seen as a form of

epistemological internationalism . It is not necessary to be African to attempt to


tell a more accurate account of the history of Europes role in the making of the
contemporary Africa and the rest of the world, for example, or to write counter-histories of the expansion of international society which
systematic barbarity of so-called Western civilisation. It is not necessary to have been colonised to recognise and document the violence, racism,

detail the

genocide and dispossession which have characterised European expansion over five hundred years.

AT: SUFFERING/VICTIM REPS K


Our use of empathy in conjunction with broader critique enables successful
movements even if it remains in local spaces like debatestar this card
Chabot Davis 4 [12/05/2004, Kimberly Chabot Davis teaches twentieth-century U.S.
literature and film at Bridgewater State College. Used to teach 20th century American literature
and culture in the History and Literature Program at Harvard University. Chabot Davis's articles
on contemporary culture have been published in Twentieth Century Literature, International
Journal of Cultural Studies, Journal for the Psychoanalysis of Culture and Society, South
Atlantic Review, Politics and Culture, and Modern Fiction Studies, Oprah's Book Club and the
politics of cross-racial empathy, International Journal of Cultural Studies 2004 7: 399]

Lauren Berlant

is right to be skeptical that a reading of a single novel could be solely responsible


for producing such radical changes in individuals. However, sympathetic reading experiences can
play an important role in a larger chain of events , alongside other moments of
critical thinking and encounters with alternative viewpoints that might shift an
individuals perspective. As the white woman Audrey said in the program on Beloved, I understand something now that I didnt
understand before, and [the film] has made me want to understand more (p. 13). Reading can clearly alter ones
sense of reality , as evidenced by a reader of Cane River who noted on the web-board: I dont know if it is
heightened awareness due to my reading the novel . . . but it seems like there is so much more in the
news about racism; just today I read two articles. Reading fiction can help a person to develop an understanding of the plight
of others and a sense of moral outrage, often seen as important precursors for action. In her ethnographic study of white womens
reading groups, sociologist Elizabeth Long argues that reading,

especially when combined with communal


reflection and discussion, provides . . . in some cases, motivation for taking individual or
collective action beyond the world of books (Long, 2003: 24). Several whites in the on-line and televised
discussions were putting their anti-racist feelings to work in the public sphere , in their jobs as
teachers and social workers serving minority communities. Although it is unlikely that fictionreading was the sole catalyst for their
occupational choices, their testimony suggests that

experiences of empathy in cultural space help to

sustain and fortify their ongoing political commitments . While Berlant doubts that
emotional shifts in the private sphere ever get converted into a larger politics of change ,
Lawrence Grossberg argues persuasively that affective relations are, at least potentially, the
condition of possibility for the optimism, invigoration, and passion which are
necessary for any struggle to change the world (Grossberg, 1992: 86). While the experience of
sympathy may produce merely self-satisfied feelings of benevolence that substitute for
committed action, I contend that the larger impediment to radical change is not sympathy itself,
but conditions that weaken its effectivity such as wide-spread public skepticism that
protest can actually accomplish social change in a world controlled by postmodern global capitalism. Like
Grossberg, I see affective culture as an underappreciated resource in combating the
disenchantment that threatens to nullify political resistance in the United States. My
work on the politics of empathetic reading contributes to a recent shift in American studies, calling for an end to the separate
spheres paradigm that divided public from private, masculine from feminine, the world of political action from the world of feeling.

The essays in the recent collection No More Separate Spheres! suggest that the line between the public and the private is a blurry
one, and that these two spheres are in fact largely imbricated (Davidson and Hatcher, 2002). With a similar goal, I

have
highlighted the political importance of empathetic reading in fostering self-transformation
and a radical interrogation of white privilege . In this particular deployment of empathy,
such moments of radical understanding could be seen as an incipient form of political action,
rather than its antithesis. This form of self-transformation operates on a continuum with
largerscale political actions in both private and public settings . Instead of equating
the political only with the arena of elections, protest movements, and collective organizing,
scholars also need to consider the importance of local, interpersonal encounters in
effecting social change . Experiencing empathy for African Americans in cultural space may move someone to object to
a racist joke among colleagues or friends, or to persuade an older relative that mixed-race marriages can produce healthy and happy
families. One of the white participants in the televised discussion of The Bluest Eye adopted three abused black girls and is
passionately working to help them to develop self-esteem. Is

her anti-racist action any less political because it


takes place within the private sphere of the family? I argue that such local and personal examples
of taking a moral stand do work to undermine racism , and are probably necessary
stepping stones for individuals to move towards more public-oriented anti-racist
acts that require greater risk . The power of culture in fostering personal selftransformation should not be undervalued . Although many of Oprahs Book Club choices have been
disparaged for their rampant emotionalism, I have argued that their solicitation of sympathy is in fact central to
their cultural power. As Larry Grossberg contends, emotive genres are politically powerful because they
provoke identification, belonging and investment, providing audiences with mattering maps
which reveal the places at which people can anchor themselves into the world, the locations of
the things that matter (1992: 82). At the end of the Oprah discussion of her book Song of Solomon, Toni Morrison revealed
her own dream to offer such mattering maps to readers: Its the dream of a writer . . . to have something important, truly
meaningful, happen to a person whos ready for the happening and the key to it is the experience of reading a book. . . . Its not a
lesson that said do this . . . and this is the solution, but to actually engage in the emotions, the actions, and the company . . . of the
characters (Howd They, 1996). Oprahs Book Club selections

do not provide solutions to social problems


concerning race and gender, but they do offer intense emotional engagement that is an
essential ingredient of political engagement. Although sympathy has often worked to
legitimate the status quo, my analysis of Oprahs Book Club demonstrates that affective reading
experiences can also disrupt ideologies of racial hierarchy . Conspicuously absent from
most analyses of cross-racial sympathy are reading experiences such as I have spotlighted here, in
which white womens empathetic encounters with African American fictional characters led to an
increased politicization and desire to combat racism in public forums such as The Oprah
Winfrey Show itself. For some white readers of African American fiction, these testimonies of suffering offer merely a
vicarious sensory experience that does little to alter their own sense of privilege. These texts produce more radical
reading effects when empathetic connections are accompanied by critical reflection, when
thought and feeling combine to result in a critique of racism and a deeper respect for
cultural difference . While Oprahs utopian claims about the power of individual texts to
change the world may seem naively optimistic, she is right about one thing reading literature and watching films
do shape the feelings and ideologies of individuals. This belief, after all, has been central to the academic study of literature, and the
motivation behind the move towards multicultural literacy in education. In

this academic climate of suspicion


toward a politics rooted in affect, critics need to consider that such cross-racial empathetic

identifications in the private sphere could play a crucial role in galvanizing support for
anti-racist public policy in America.

Students in academic spaces can formulate responses to suffering are good --taking a particular stand is key
Kim Sawchuk 2, PhD, Social and Political Thought, York University, prof of comm at Concordia,
editor of the Canadian Journal of Communication, general book review of Distant Suffering by Luc
Boltanski, Canadian journal of Communication Vol 27, No 1 (2002)
Inherent in the politics of pity in the modern period is the problem of dealing with suffering from a distance
and the "massification of a collection of unfortunates who are not there in person" (p. 13). Although contemporary media may have
"dramatized" the spectacle of distant suffering in the past 30 years, they neither invented nor
caused this condition. Historical examples also bolster Boltanski's claim that the media did not inaugurate the politics of pity - rather, its logic was set
out more than 200 years ago. Boltanski carefully examines this logic and the paradoxes it creates in the book's three sections. Part 1 lays out the argument. Part 2 relies heavily
on literary sources to analyze the "topos," a term he borrows from rhetoric, of the idea of pity and suffering. The third section deals with the question of pity and misfortune,
drawing primarily on historical and contemporary examples, such as the work of Doctors Without Borders and the clash in the late 1950s between Sartre and Merleau-Ponty.
Each chapter is replete with insight, making this a difficult book to summarize. Every word and every argument is so intricately intertwined with the next that paraphrasing
seems a travesty. The third section should be of interest to those located in the disciplines of communications or media studies. Here it is important to recall the subtitle of the

Boltanski returns to the question of the spectator and the anxieties of those
who wish to do something about what they see unfolding on their screens . He asks: "[H]ow might the contemporary
book, Morality, the Media and Politics.

spectators' anxiety be reduced without averting their gaze from misfortune or by abandoning the project inherent in the modern definition of politics of facing up to unnecessary

What could political action be, given the fact that suffering does occur at a
distance and that not every struggle can be taken on with equal commitment ? First, he argues that there
suffering and relieving it[?]" (p. 159).

is a political, technical, and moral necessity to open up a discussion of


commitment and ideology, although what he means by ideology is not adequately
explained. Second, he contends that witnessing suffering means that morally we are asked to act. Commitment is commitment
to some kind of action . Third, he promotes the idea that speech is action. "One can
commit oneself through speech; by taking a stance, even when alone, of someone
who speaks to somebody else about what they have seen" (p. xv). By speaking - to
others and even to oneself - we recognize and acknowledge that speech must be understood
as a form of action (p. 154). One of the conditions of Boltanski's argument is a clear distinction between the world of representation and the world of
action. He writes: "Informed by representation, words must really be deployed in the world of action in order to be effective" (p. 154).
deconstructionist criticism, primarily meaning the writings of Jean

He is critical of

Baudrillard, which blurs this distinction to too

great an extreme , thereby "holding the order of action" at arm's length or making it
illusionary (p. 154). He contends that this position makes the very intention to act nothing but a nave
illusion creating an "empire of suspicion" (p. 158). Boltanski does not claim that we remain without an emotional commitment to causes, but
rather that "to prevent the unacceptable drift of emotions close towards the fictional we must maintain an orientation towards
action, a disposition to act, even if this is only by speaking out in support of the
unfortunate" (p. 153). What then are the properties of effective speech? Boltanski turns to phenomenology and semantics, concluding that effective speech
involves: (a) intentionality; (b) incorporation in bodily gestures and movements; (c) sacrifice of other possible actions; (d) the presence of others; and, (e) a commitment (p.
185). Intentionality involves an intention to speak meaningfully, not just engage in idle chatter. Action and intention are connected to each other in effective action realized in
the world. Intention incorporated in action is "expression." This kind of expressive political speech must involve risk for spectators - they may be chastized, they may be
contested, or they may be at physical risk in authoritarian regimes. Boltanski goes on to classify different types of action as strong and weak, collective and individual. He builds
an argument for local chapters of groups supporting humanitarian movements, such as Amnesty International, for they enable one to avoid the alternative of either on-the-spot
involvement or distant spectacle. They are one way to breach the schism between abstract universalism and communitarian withdrawal: "The humanitarian claim for more or
less distant causes can thus avoid the alternative of abstract universalism - easily accused of being fired up for distant suffering the better to avert its eyes from those close at
hand - or of communitarian withdrawal into itself - which only attends to misfortune when it affects those nearest - by being rooted in groups and thereby linked to preexisting
solidarities and local interests" (p. 190). In other words, expression is most "authentic" for Boltanski when made manifest in actions, like participating in a demonstration or

protest, which incarnates our beliefs and displays our commitments. By incorporating an action, the person communicates an observable tendency. But is this enough?
Boltanski is

concerned by apathy and asks us to consider that we are doomed, inevitably, to imperfection in our politics. Despite this, we must

make the attempt to be "moral subjects" - that is, committed and engaged subjects. Because he recognizes the
difficulties of negotiating these contradictions, he avoids moralizing. He is no Habermasian trying to outline the conditions for an ideal-speech situation. In Boltanski's book,

we live in imperfect worlds and we must contend with this. He asks that we resurrect
compassion into our politics , which he says is always particular and practical, as it is
oriented toward doing something about a situation . Unlike pity, it engages with the person suffering . But pity isn't
always a bad thing in this analysis. Pity generalizes in order to deal with distance, and in so doing one may discover emotion and feeling for others that may translate into speech

A spectacle of suffering may end with a commitment to involvement. Boltanski realizes the challenge, yet
remains optimistic that humans are capable of such a move . There are, as he notes, an "excess of unfortunates" in our world.
or action.

The problem remains to whom we extend aid or pity, given their great numbers (p. 155). This is true both in the realm of action, but also in the realm of representation. So many
people are suffering and there is not enough media space for them all (p. 155). Boltanski does not prioritize causes or instances of grief. He does, however, suggest that the media
represent any unfortunate groups taking action to confront and escape their distress. It is unethical to only depict them in the passive act of suffering (p. 190). He acknowledges
that the mediatization of suffering may incite action. For example, it may protect populations against their own rulers, if only temporarily, for such depictions do not necessarily
change the internal political situation. His analysis assumes that spectators, who are

democratic citizens, have a role to play in

lobbying and pressuring their own governments to take action

(p. 184). Again, while aware that public

opinion may be manipulated, he argues that public-opinion polls are powerful tools. Answering a poll is depicted as a potentially effective form of speech and an "adequate
response to the call for action" (p. 185). Distant Suffering thus describes, in sometimes painful detail, a

wavering between selfish egoism and

altruistic commitment to causes. Boltanski describes how we may, unfortunately, cultivate ourselves by becoming absorbed in our own pity when
looking at the spectacle of someone else's suffering, a phenomenon that has been far too present since the September 11 bombing of the World Trade Center in New York.
Boltanski

tries to lead us out of this self-absorption into the world of effective political

action by offering a range of involvement. While advocating commitment and debates about morality as part of the solution, this
is no smug celebration of the "return to kindness" or an easy denunciation of the perverse delight of spectacles of suffering. In considering distant suffering as the "logical
consequence" of the introduction of pity into politics over 200 years ago, we are asked to concern ourselves with the present. Boltanski ends his fine treatise by exhorting us to

To be concerned with the present is no


small matter. For over the past, ever gone by, and over the future, still non-existent, the present has an overwhelming privilege:
that of being real" (p. 192). Naive? Perhaps. Boltanski does not provide simple or quick answers to the dilemma, but leaves one with the hope that pity might
lead to compassion, commitment, and social change - even if such measures do not end
all suffering once and for all. As such, this translation from the original French text is a welcome addition to contemporary debates in political
quit looking to past injustices, to stop anticipating future injustice, and to stay focused on the present. "

communication.

Permutation: vote affirmative and refuse using our narratives suffering for
liberalismthere is nothing intrinsically bad about our representationsdont
throw out the baby with the bathwater
Chabot Davis 4
[12/05/2004, Kimberly Chabot Davis teaches twentieth-century U.S. literature and film at
Bridgewater State College. Used to teach 20th century American literature and culture in the
History and Literature Program at Harvard University. Chabot Davis's articles on contemporary
culture have been published in Twentieth Century Literature, International Journal of Cultural
Studies, Journal for the Psychoanalysis of Culture and Society, South Atlantic Review, Politics
and Culture, and Modern Fiction Studies, Oprah's Book Club and the politics of cross-racial
empathy, International Journal of Cultural Studies 2004 7: 399]

other cultural critics


and race theorists treat sympathy as an inherently colonizing action, and they reduce empathy to
an imperialistic drive to incorporate the other into the self. While some of these critics are informed by
While some object to the sympathetic emotions on the grounds that they substitute for political action,

postcolonial theories exposing the drive for knowledge of the exotic other as a form of power, others

are influenced by the


Freudian view of identification as a hostile erasure of the other, in which the object that we long
for and prize is assimilated by eating and is in that way annihilated as such (Freud, 1955: 105; see also
hooks, 1992). In critical race theory, studies of cross-racial sympathy take a decidedly pessimistic tone about the possibilities of
coalition, which has the unfortunate consequence of reifying the very racial categories that they seek to undermine.10 For example,
Robyn Wiegman despairs that integrationist novels and films are ultimately wedded to white authority, and concludes that the
transformatory hope of identifying with the pain and suffering of others seems ever more bound to an imperialistic cast (1995: 200).
In researching her book Racechanges, Susan Gubar sadly concludes that crossracial masquerade and imitation inevitably leads to
the disappearance of the others otherness (1997: 245). While these writers take a despairing tone, Doris Sommer aggressively
condemns cross-racial sympathy and empathy alike. Focusing on the narrative strategies that ethnic writers use to refuse access to
white readers, Sommer dismisses the sympathy of white liberals as an appropriation in the guise of an embrace (1994: 543) and a
facile form of connection that lasts hardly longer than the reading of a novel (p. 529). Arguing that empathy is the egocentric
energy that drives one subject to impersonate another (1999: 22), she echoes Freuds view of identification as a metaphoric
substitution of the self for the other. These writers shed light on a problematic possibility of sympathy and empathy that the
privileged sympathizer will ignore differences in his or her zeal to connect emotionally with the sufferer. Erasing the subjective
experience of people of color, the white empathizer falsely claims someone elses particular pain as his own. While some

critics
focus on this desire for sameness as an erasure, others are more critical of the hierarchy they
believe is implicit in sympathys operation. As Berlant puts it, compassion is a term denoting
privilege: the sufferer is over there (2004: 4) and the observer has the power to either help or turn
away. I agree that sympathy may involve power relations between subject and object, and may keep hierarchies firmly in place by
granting the sympathizer a feeling of benevolent largesse and by denying agency to the sufferer. These consequences,
however, are not implicit to the operation of sympathetic emotions . Empathetic
experiences of seeing from the vantage point of another can lead to a recognition of that
persons subjecthood and agency , and can lead the white empathizer to not only
become critically aware of racial hierarchy , but to desire to work against the
structures of inequality wherein her own power resides.

My case study of Oprahs Book Club

addresses both the promise and limitations of empathy and sympathy, asserting that the

politics of these emotions


depend upon how they are experienced and to what end they are employed . While the

possibility of appropriation is an important concern, critics

also have a responsibility to bring to light

moments of empathy or compassion with progressive political significance, lest


we lose hope in the potential for change in the racial order . It strikes me as particularly ironic
that these despairing conclusions are voiced by white anti-racist scholars whose own critical work attests to the radical potential of
some acts of cross-racial affiliation. In

our zeal to avoid celebratory analyses that underestimate the power


of white hegemony, we need to avoid throwing out the proverbial baby (empathy and
compassion) with the bathwater. As many social scientists have argued, empathy can inhibit
aggression, and the absence of empathy is a telling feature of inter-group violence , such as the
Israeli/Palestinian conflict.11 In the context of an alarming international rise in hate groups and terrorism , left-oriented
scholars cannot afford to give up on empathys promise of fostering cross-cultural
understanding and desires for social justice and equality. Reception analysis My reception study
suggests that sympathys

colonizing functions and its ability to inhibit action are not intrinsic to

its structure , but merely one possible deployment thereof. By offering evidence here of the more radical
possibilities of empathy visible on The Oprah Winfrey Show, I do not claim that the critics of sympathy are categorically
wrong, but merely that they are diminishing the complexity of the sympathetic emotions . One could argue
that a similarly one-sided argument is put forth by Martha Nussbaum (2001), who optimistically implies
that compassion is the cure for what ails modern society . Predicated on the assumption that reading good
books produces empathetic and moral people, Nussbaums philosophical reflections ignore the considerable diversity of reader

responses to texts. My reception study, however, exposes both negative and positive strains of empathy and sympathy, while
drawing attention to previously underappreciated progressive effects. While I do not claim that these radical deployments are
predominant, either on Oprahs programs or in the reading public at large, I found enough recurrence to warrant attention.

Their victimization arguments are elitistintellectually positing an abstract


situation where we identify someone as a victim which thereby re-entrenches their
statusour argument is not that people who need water are merely victims but
that they are OUR victims that are hungry because we have access to water and
because of this fact our acts towards them are reparations for a history of
exploitation and violencetheir victimization arguments fall apart in this context
Ruiz and Minguez 1 [Prof. Dr Pedro Ortega Ruiz, Facultad de Educacio n, Campus de
Espinardo, Universidad de Murcia, Global Inequality and the Need for Compassion: issues in
moral and political education Journal of Moral Education, Vol. 30, No. 2, 2001]

It could be said that there are demands for justice because there are victims who recognise
themselves as such or are so recognised by others. Their suffering is felt as an affront to their dignity, something
which should not be. The starting point of morality is the shout, sometimes muffled, of those who
suffer, of the oppressed and excluded, who in this condition find themselves outside the
institutional framework where others can defend their interests and moral hopes or voice their
criticism against the conditions of inequality. To hear the beseechings which come from the
others suffering is to go beyond a paternalistic piety. The evidence of suffering requires the mediation of an
analysis of the causes of suffering and requires the establishment of an asymmetry in the face of the overriding inequality of the
victim. The

victim must be granted an authority which breaks with the formal framework of
equality of opportunities within which his exclusion has been produced. Compassion is not an
alternative to justice but rather the source of a new understanding of justice, since continuing to
recognise the same rights for all may seem correct from a formal viewpoint, but from a realistic
perspective implies the perpetuation of that situation which causes victims and their suffering.

SECURITY ANSWERS

2AC INSTITUTIONS GOOD


Institutions matter their description of our aff doesnt assume the constant
reflexivity that the aff would create in decision-making processes towards certain
risks
Jeremi Suri & Benjamin Valentino 16 Jeremi is a Distinguished Professor for Global
Leadership, History, and Public Policy, University of Texas at Austin and Benjamin is an
Associate Professor of Government, Dartmouth College (Sustainable Security: Rethinking
American National Security Strategy, 2016,
http://tobinproject.org/sites/tobinproject.org/files/assets/Sustainable%20Security%20-%20All
%20Chapters.pdf) hk
institutions matter more than policymakers often realize . The organizations that
govern American representatives abroad must become more flexible, focused, and efficient to
better match constrained resources with ambitious policy aims. John Hall argues that creating a culture of
innovation within the armed services is crucial, particularly among military officers rising through the ranks . This requires
more emphasis on free-thinking, and even dissent, within military education and promotion boards. William
Inboden observes that the national security decision-making process, frequently reformed, requires
continual readjustment to ensure that presidents get the kinds of advice they need for the
challenges of their time. The National Security Council (and its associated bodies) has often lacked creativity because it
Fifth,

expends too much energy on turf battles and gives too little attention to integrating different perspectives into a coherent policy
discussion among principals. The military, the NSC, and other

US security organizations must learn to

embrace change and controlled risk-taking , rather than the slow moving accumulation of
claims on resources that have characterized past bureaucratic battles. Leaner institutions can be
more effective and sustainable.

Their framework/alternative arguments distract from institutional analysis that


undermines their sustainability and effectiveness thats net offence
Welsh, Comm Prof @ Appalachia State University, 13
(Scott, The Rhetorical Surface of Democracy: How Deliberative Ideals Undermine Democratic
Politics, pg. 81)
Certeau does not, however, reduce political contest to the "micro" politics of everyday life. Just

because he recognizes that the


power of the weak resides in the effective authority of the strong does not mean that we
can rest assured that everything will simply work out in the end.

Jodi Dean

finds a naive

faith of this kind in those who imagine that accelerated and broadly dispersed
conversations , made possible by the Internet and social media, might ultimately deliver free
and equal democratic deliberation.

So in response, she argues that such

efforts "to displace polities

onto the activities of everyday or ordinary people" draw attention away from the
still very real need for political challengers to build support , win elections, and

ultimately exercise authority themselves . While Certeau is partly responsible for drawing recent scholarly
attention to the politics of everyday life, he never- theless deliberately avoids falling into the trap Dean identifies. Instead, as
important as the daily "capture of speech" is to democratic politics, we must not, Ceneau argues, attribute a magical democratic
power to vernacular voices. Speech, he clarifies, is

not an adequate "substitute" for "work." S2 Certeau's


distinction between speech and work, however, may be either more or less than meets the eye. It signals, rather, the
necessity of a sustained capture of speech , which is only possible upon winning a
measure of institutional authority . Hence, Certeau rejects the "jubilation" of intellectuals
"who wish to collapse a system of authority without preparing for its replacement." 83
Instead, meaningful change requires forms of symbolic resistance that plan an eventual campaign
for and a reintegration with state authority . He writes, "If it is not organized, if it is not inscribed, ewen as a
strategy, within the network of national forces in order to effec'ively change a system, (al demand Of conscience Will be neither
reformist nor revolutionary," but Will be ' 'extinguished.'

2AC NO SPILLOVER
One speech act doesnt cause securitization its an ongoing process

Ghughunishvili 10
Securitization of Migration in the United States after 9/11: Constructing Muslims and Arabs as
Enemies Submitted to Central European University Department of International Relations
European Studies In partial fulfillment of the requirements for the degree of Master of Arts
Supervisor: Professor Paul Roe http://www.etd.ceu.hu/2010/ghughunishvili_irina.pdf

As provided by the Copenhagen School securitization theory is comprised by speech act , acceptance
of the audience and facilitating conditions or other non-securitizing actors contribute to a successful securitization. The
causality or a one-way relationship between the speech act, the audience and securitizing actor,
where politicians use the speech act first to justify exceptional measures, has been criticized by
scholars, such as Balzacq. According to him, the one-directional relationship between the three factors, or some of them, is not the
best approach. To

fully grasp the dynamics, it will be more beneficial to rather than looking for a
one-directional relationship between some or all of the three factors highlighted, it could be
profitable to focus on the degree of congruence between them. 26 Among other aspects of the Copenhagen
Schools theoretical framework, which he criticizes, the thesis will rely on the criticism of the lack of context and the rejection of a
one-way causal relationship between the audience and the actor. The

process of threat construction, according to


him, can be clearer if external context, which stands independently from use of language, can be
considered. 27 Balzacq opts for more context-oriented approach when it comes down to securitization through the speech act,
where a single speech does not create the discourse, but it is created through a long process, where
context is vital. 28 He indicates: In reality, the speech act itself, i.e. literally a single security
articulation at a particular point in time, will at best only very rarely explain the entire social
process that follows from it. In most cases a security scholar will rather be confronted with a
process of articulations creating sequentially a threat text which turns sequentially into a
securitization. 29 This type of approach seems more plausible in an empirical study, as it is more likely that a single
speech will not be able to securitize an issue, but it is a lengthy process, where a the audience
speaks the same language as the securitizing actors and can relate to their speeches .

2AC PERMUTATION
Permutation do the plan and interrogate the 1acs epistemological failures
those failures are inevitable and can be interrogated endlessly but if
implementing the aff can still be a good idea some failures, voting for the perm
enables transformative potential
Nunes 12 [Reclaiming the political: Emancipation and critique in security studies, Joo Nunes,
Security Dialogue 2012 43: 345, Politics and International Studies, University of Warwick, UK,
p. sage publications]

In the works of these authors, one can identify a tendency to see security as inherently
connected to exclusion, totalization and even violence. The idea of a logic of security is now
widely present in the critical security studies literature. Claudia Aradau (2008: 72), for example,
writes of an exclusionary logic of security underpinning and legitimizing forms of domination.
Rens van Munster (2007: 239) assumes a logic of security, predicated upon a political
organization on the exclusionary basis of fear. Laura Shepherd (2008: 70) also identifies a
liberal and highly problematic organizational logic in security. Although there would probably
be disagreement over the degree to which this logic is inescapable, it is symptomatic of an
overwhelmingly pessimistic outlook that a great number of critical scholars are now making the
case for moving away from security. The normative preference for desecuritization has been
picked up in attempts to contest, resist and unmake security (Aradau, 2004; Huysmans, 2006;
Bigo, 2007). For these contributions, security cannot be reconstructed and political
transformation can only be brought about when security and its logic are removed from the
equation (Aradau, 2008; Van Munster, 2009; Peoples, 2011). This tendency in the literature is
problematic for the critique of security in at least three ways. First, it constitutes a blind spot
in the effort of politicization. The assumption of an exclusionary, totalizing or violent logic of
security can be seen as an essentialization and a moment of closure . To be faithful to itself,
the politicization of security would need to recognize that there is nothing natural or necessary
about security and that security as a paradigm of thought or a register of meaning is also a
construction that depends upon its reproduction and performance through practice. The
exclusionary and violent meanings that have been attached to security are themselves the result
of social and historical processes, and can thus be changed. Second, the institution of this
apolitical realm runs counter to the purposes of critique by foreclosing an engagement with
the different ways in which security may be constructed. As Matt McDonald (2012) has argued,
because security means different things for different people, one must always understand
it in context . Assuming from the start that security implies the narrowing of choice and the
empowerment of an elite forecloses the acknowledgment of security claims that may seek to
achieve exactly the opposite: alternative possibilities in an already narrow debate and the
contestation of elite power.5 In connection to this, the claims to insecurity put forward by
individuals and groups run the risk of being neglected if the desire to be more secure is
identified with a compulsion towards totalization, and if aspirations to a life with a degree of

predictability are identified with violence. Finally, this tendency blunts critical security
studies as a resource for practical politics . By overlooking the possibility of
reconsidering security from within opting instead for its replacement with other ideals
the critical field weakens its capacity to confront head-on the exceptionalist connotations that
security has acquired in policymaking circles. Critical scholars run the risk of playing into this
agenda when they tie security to exclusionary and violent practices, thereby failing to question
security actors as they take those views for granted and act as if they were inevitable. Overall,
security is just too important both as a concept and as a political instrument to be simply
abandoned by critical scholars. As McDonald (2012: 163) has put it, If security is politically
powerful, is the foundation of political legitimacy for a range of actors, and involves the
articulation of our core values and the means of their protection, we cannot afford to allow
dominant discourses of security to be confused with the essence of security itself. In sum, the
trajectory that critical security studies has taken in recent years has significant limitations .
The politicization of security has made extraordinary progress in problematizing predominant
security ideas and practices; however, it has paradoxically resulted in a depoliticization of the
meaning of security itself. By foreclosing the possibility of alternative notions of security, this
imbalanced politicization weakens the analytical capacity of critical security studies, undermines
its ability to function as a political resource and runs the risk of being politically
counterproductive . Seeking to address these limitations, the next section revisits
emancipatory understandings of security.

2AC REALISM
The AFF doesnt change the framework states operate within that takes out all of
their root cause claims, external impacts, and justifies our epistemology their
heuristic makes war and structural violence more likely
de Araujo, professor for Ethics at Universidade do Estado do Rio de Janeiro, 14
(Marcelo, Moral Enhancement and Political Realism, Journal of Evolution and Technology
24(2): 29-43)
moral enhancement theorists argue that a society of morally enhanced individuals would
be in a better position to cope with important problems that humankind is likely to face in the future
such as, for instance, the threats posed by climate change , grand scale terrorist attacks , or the risk of catastrophic
Some

wars. The assumption here is quite simple: our inability to cope successfully with these problems
stems mainly from a sort of deficit in human beings moral motivation . If human beings were
morally better if we had enhanced moral dispositions there would be fewer wars, less terrorism , and more
willingness to save our environment . Although simple and attractive, this assumption is , as I intend to show,
false . At the root of threats to the survival of humankind in the future is not a deficit in our
moral dispositions, but the endurance of an old political arrangement that prevents the
pursuit of shared goals on a collective basis. The political arrangement I have in mind here is the international system of states. In my
analysis of the political implications of moral enhancement, I intend to concentrate my attention only on the supposition that we could avoid major wars in the future by making
individuals morally better. I do not intend to discuss the threats posed by climate change, or by terrorism, although some human enhancement theorists also seek to cover these
topics. I will explain, in the course of my analysis, a conceptual distinction between human nature realism and structural realism, well-known in the field of international
relations theory. Thomas Douglas seems to have been among the first to explore the idea of moral enhancement as a new form of human enhancement. He certainly helped to
kick off the current phase of the debate. In a paper published in 2008, Douglas suggests that in the future people might use biomedical technology to morally enhance
themselves. Douglas characterizes moral enhancement in terms of the acquisition of morally better motives (Douglas 2008, 229). Mark Walker, in a paper published in 2009,
suggests a similar idea. He characterizes moral enhancement in terms of improved moral dispositions or genetic virtues: The Genetic Virtue Program (GVP) is a proposal for
influencing our moral nature through biology, that is, it is an alternate yet complementary means by which ethics and ethicists might contribute to the task of making our lives
and world a better place. The basic idea is simple enough: genes influence human behavior, so altering the genes of individuals may alter the influence genes exert on behavior.
(Walker 2009, 2728) Walker does not argue in favor of any specific moral theory, such as, for instance, virtue ethics. Whether one endorses a deontological or a utilitarian
approach to ethics, he argues, the concept of virtue is relevant to the extent that virtues motivate us either to do the right thing or to maximize the good (Walker 2009, 35). Moral
enhancement theory, however, does not reduce the ethical debate to the problem of moral dispositions. Morality also concerns, to a large extent, questions about reasons for
action. And moral enhancement, most certainly, will not improve our moral beliefs; neither could it be used to settle moral disagreements. This seems to have led some authors
to criticize the moral enhancement idea on the ground that it neglects the cognitive side of our moral behavior. Robert Sparrow, for instance, argues that, from a Kantian point of
view, moral enhancement would have to provide us with better moral beliefs rather than enhanced moral motivation (Sparrow 2014, 25; see also Agar 2010, 74). Yet, it seems to

Many people, across different countries, already share


moral beliefs relating, for instance, to the wrongness of harming or killing other people arbitrarily, or to
me that this objection misses the point of the moral enhancement idea.

the moral requirement to help people in need. They may share moral beliefs while not sharing the same reasons for these beliefs, or perhaps even not being able to articulate the
beliefs in the conceptual framework of a moral theory (Blackford 2010, 83). But although they share some moral beliefs, in some circumstances they may lack the appropriate
motivation to act accordingly. Moral enhancement, thus, aims at improving moral motivation, and leaves open the question as to how to improve our moral judgments. In a
recent paper, published in The Journal of Medical Ethics, neuroscientist Molly Crockett reports the state of the art in the still very embryonic field of moral enhancement. She
points out, for example, that the selective serotonin reuptake inhibitor (SSRI) citalopram seems to increase harm aversion. There is, moreover, some evidence that this
substance may be effective in the treatment of specific types of aggressive behavior. Like Douglas, Crockett emphasizes that moral enhancement should aim at individuals moral
motives (Crockett 2014; see also Spence 2008; Terbeck et al. 2013). Another substance that is frequently mentioned in the moral enhancement literature is oxytocin. Some

studies suggest that willingness to cooperate with other people,and to trust unknown prospective cooperators, may be
enhanced by an increase in the levels of oxytocin in the organism (Zak 2008, 2011; Zak and Kugler 2011; Persson and Savulescu 2012,
118119). Oxytocin has also been reported to be associated with the subjective experience of empathy (Zak 2011, 55; Zak and Kugler 2011, 144). The question I would like to
examine now concerns the supposition that moral enhancement comprehended in these terms and assuming for the sake of argument that, some day, it might become

The assumption that there is a relationship


between, on the one hand, threats to the survival of humankind and, on the other, a sort of deficit in our
moral dispositions is clearly made by some moral enhancements theorists. Douglas, for instance, argues that according to
effective and safe may also help us in coping with the threat of devastating wars in the future.

many plausible theories, some of the worlds most important problems such as developing world poverty, climate change and war can be attributed to these moral deficits
(2008, 230). Walker, in a similar vein, writes about the possibility of using biotechnology to alter our biological natures in an effort to reduce evil in the world (2009, 29). And
Julian Savulescu and Ingmar Persson go as far as to defend the the need for moral enhancement of humankind in a series of articles, and in a book published in 2012. One of
the reasons Savulescu and Persson advance for the moral enhancement of humankind is that our moral dispositions seem to have remained basically unchanged over the last
millennia (Persson and Savulescu 2012, 2). These dispositions have proved thus far quite useful for the survival of human beings as a species. They have enabled us to cooperate
with each other in the collective production of things such as food, shelter, tools, and farming. They have also played a crucial role in the creation and refinement of a variety of

human institutions such as settlements, villages, and laws. Although the possibility of free-riding has never been fully eradicated, the benefits provided by cooperation have
largely exceeded the disadvantages of our having to deal with occasional uncooperative or untrustworthy individuals (Persson and Savulescu 2012, 39). The problem, however, is
that the same dispositions that have enabled human beings in the past to engage in the collective production of so many artifacts and institutions now seem powerless in the face
of the human capacity to destroy other human beings on a grand scale, or perhaps even to annihilate the entire human species. There is, according to Savulescu and Persson, a
mismatch between our cognitive faculties and our evolved moral attitudes: [] as we have repeatedly stressed, owing to the progress of science, the range of our powers of
action has widely outgrown the range of our spontaneous moral attitudes, and created a dangerous mismatch (Persson and Savulescu 2012, 103; see also Persson and Savulescu
2010, 660; Persson and Savulescu 2011b; DeGrazie 2012, 2; Raki 2014, 2). This worry about the mismatch between, on the one hand, the modern technological capacity to
destroy and, on the other, our limited moral commitments is not new. The political philosopher Hans Morgenthau, best known for his defense of political realism, called
attention to the same problem nearly fifty years ago. In the wake of the first successful tests with thermonuclear bombs, conducted by the USA and the former Soviet Union,
Morgenthau referred to the contrast between the technological progress of our age and our feeble moral attitudes as one of the most disturbing dilemmas of our time: The first
dilemma consists in the contrast between the technological unification of the world and the parochial moral commitments and political institutions of the age. Moral
commitments and political institutions, dating from an age which modern technology has left behind, have not kept pace with technological achievements and, hence, are
incapable of controlling their destructive potentialities. (Morgenthau 1962, 174) Moral enhancement theorists and political realists like Morgenthau, therefore, share the thesis
that our natural moral dispositions are not strong enough to prevent human beings from endangering their own existence as a species. But they differ as to the best way out of
this quandary: moral enhancement theorists argue for the re-engineering of our moral dispositions, whereas Morgenthau accepted the immutability of human nature and
argued, instead, for the re-engineering of world politics. Both positions, as I intend to show, are wrong in assuming that the dilemma results from the weakness of our

both positions are correct in


recognizing the real possibility of global catastrophes resulting from the malevolent use of,
spontaneous moral dispositions in the face of the unprecedented technological achievements of our time. On the other hand,

for instance, biotechnology or nuclear capabilities.

The supposition that individuals unwillingness to cooperate with each

other, even when they would be better-off by choosing to cooperate, results from a sort of deficit of dispositions such as altruism, empathy, and benevolence has been at the core
of some important political theories. This idea is an important assumption in the works of early modern political realists such as Machiavelli and Thomas Hobbes. It was also
later endorsed by some well-known authors writing about the origins of war in the first half of the twentieth century. It was then believed, as Sigmund Freud suggested in a text

Freud went as far


as to suggest that human beings have an ingrained inclination to aggression and
destruction (Aggressionstrieb, Aggressionsneigung, and Destruktionstrieb), and that this inclination has a good biological basis (biologisch wohl begrndet)
(Freud 1999, 2024; see also Freud 1950; Forbes 1984; Pick 1993, 211227; Medoff 2009). The attempt to employ Freuds conception of
human nature in understanding international relations has recently been resumed, for instance by Kurt
from 1932, that the main cause of wars is a human tendency to hatred and destruction (in German: ein Trieb zum Hassen und Vernichtung).

Jacobsen in a paper entitled Why Freud Matters: Psychoanalysis and International Relations Revisited, published in 2013. Morgenthau himself was deeply influenced by
Freuds speculations on the origins of war.1 Early in the 1930s, Morgenthau wrote an essay called On the Origin of the Political from the Nature of Human Beings (ber die
Herkunft des Politischen aus dem Wesen des Menschen), which contains several references to Freuds theory about the human propensity to aggression.2 Morgenthaus most
influential book, Politics among Nations: The Struggle for Power and Peace, first published in 1948 and then successively revised and edited, is still considered a landmark work
in the tradition of political realism. According to Morgenthau, politics is governed by laws that have their origin in human nature: Political realism believes that politics, like
society in general, is governed by objective laws that have their roots in human nature (Morgenthau 2006, 4). Just like human enhancement theorists, Morgenthau also takes
for granted that human nature has not changed over recent millennia: Human nature, in which the laws of politics have their roots, has not changed since the classical
philosophies of China, India, and Greece endeavored to discover these laws (Morgenthau 2006, 4). And since, for Morgenthau, human nature prompts human beings to act
selfishly, rather than cooperatively, political leaders will sometimes favor conflict over cooperation, unless some superior power compels them to act otherwise. Now, this is
exactly what happens in the domain of international relations. For in the international sphere there is not a supranational institution with the real power to prevent states from
pursuing means of self-defense. The acquisition of means of self-defense, however, is frequently perceived by other states as a threat to their own security. This leads to the
security dilemma and the possibility of war. As Morgenthau put the problem in an article published in 1967: The actions of states are determined not by moral principles and
legal commitments but by considerations of interest and power (1967, 3). Because Morgenthau and early modern political philosophers such as Machiavelli and Hobbes
defended political realism on the grounds provided by a specific conception human nature, their version of political realism has been frequently called human nature realism.
The literature on human nature realism has become quite extensive (Speer 1968; Booth 1991; Freyberg-Inan 2003; Kaufman 2006; Molloy 2006, 8285; Craig 2007;
Scheuerman 2007, 2010, 2012; Schuett 2007; Neascu 2009; Behr 2010, 210225; Brown 2011; Jtersonke 2012). It is not my intention here to present a fully-fledged account of
the tradition of human nature realism, but rather to emphasize the extent to which some moral enhancement theorists, in their description of some of the gloomy scenarios
humankind is likely to face in the future, implicitly endorse this kind of political realism. Indeed, like human nature realists, moral enhancement theorists assume that human
nature has not changed over the last millennia, and that violence and lack of cooperation in the international sphere result chiefly from human natures limited inclination to
pursue morally desirable goals. One may, of course, criticize the human enhancement project by rejecting the assumption that conflict and violence in the international domain

Sparrow correctly argues that


structural issues, rather than human nature , constitute the main factor underlying
political conflicts (Sparrow 2014, 29). But he does not explain what exactly these structural issues are, as I intend to do later. Sparrow is right in rejecting the
should be explained by means of a theory about human nature. In a reply to Savulescu and Persson,

human nature theory underlying the human enhancement project. But this underlying assumption, in my view, is not trivially false or simply ludicrous, as he suggests. Human
nature realism has been implicitly or explicitly endorsed by leading political philosophers ever since Thucydides speculated on the origins of war in antiquity (Freyberg-Inan
2003, 2336). True, it might be objected that human nature realism, as it was defended by Morgenthau and earlier political philosophers, relied upon a metaphysical or
psychoanalytical conception of human nature, a conception that, actually, did not have the support of any serious scientific investigation (Smith 1983, 167). Yet, over the last few
years there has been much empirical research in fields such as developmental psychology and evolutionary biology that apparently gives some support to the realist claim. Some
of these studies suggest that an inclination to aggression and conflict has its origins in our evolutionary history. This idea, then, has recently led some authors to resume human
nature realism on new foundations, devoid of the metaphysical assumptions of the early realists, and entirely grounded in empirical research. Indeed, some recent works in the
field of international relations theory already seek to call attention to evolutionary biology as a possible new start for political realism. This point is clearly made, for instance, by
Bradley Thayer, who published in 2004 a book called Darwin and International Relations: On the Evolutionary Origins of War and Ethnic Conflict. And in a paper published in
2000, he affirms the following: Evolutionary theory provides a stronger foundation for realism because it is based on science, not on theology or metaphysics. I use the theory to
explain two human traits: egoism and domination. I submit that the egoistic and dominating behavior of individuals, which is commonly described as realist, is a product of
the evolutionary process. I focus on these two traits because they are critical components of any realist argument in explaining international politics. (Thayer 2000, 125; see also
Thayer 2004) Thayer basically argues that a tendency to egoism and domination stems from human evolutionary history. The predominance of conflict and competition in the
domain of international politics, he argues, is a reflex of dispositions that can now be proved to be part of our evolved human nature in a way that Morgenthau and other earlier
political philosophers could not have established in their own time. Now, what some moral enhancement theorists propose is a direct intervention in our evolved limited moral
psychology as a means to make us fit to cope with some possible devastating consequences from the predominance of conflict and competition in the domain of international
politics (Persson and Savulescu 2010, 664). Moral enhancement theorists comprehend the nature of war and conflicts, especially those conflicts that humankind is likely to face
in the future, as the result of human beings limited moral motivations. Compared to supporters of human nature realism, however, moral enhancement theorists are less
skeptical about the prospect of our taming human beings proclivity to do evil. For our knowledge in fields such as neurology and pharmacology does already enable us to
enhance peoples performance in a variety of activities, and there seems to be no reason to assume it will not enable us to enhance people morally in the future. But

question, of course, is whether moral enhancement will also improve the prospect of our coping

the

successfully with some major threats to the survival of humankind , as Savulescu and Persson propose, or to reduce evil
in the world , as proposed by Walker. V. The point to which I would next like to call attention is that human nature realism which is implicitly presupposed by
some moral enhancement theorists has been much criticized over the last decades within the tradition of political realism itself. Structural realism, unlike human nature
realism, does not seek to derive a theory about conflicts and violence in the context of international relations from a theory of the moral shortcomings of human nature.
Structural realism was originally proposed by Kenneth Waltz in Man, the State and War, published in 1959, and then later in another book called Theory of International
Politics, published in 1979. In both works, Waltz seeks to avoid committing himself to any specific conception of human nature (Waltz 2001, xxi). Waltzs thesis is that the
thrust of the political realism doctrine can be retained without our having to commit ourselves to any theory about the shortcomings of human nature. What is relevant for our
understanding of international politics is, instead, our understanding of the structure of the international system of states (Waltz 1986). John Mearsheimer, too, is an
important contemporary advocate of political realism. Although he seeks to distance himself from some ideas defended by Waltz, he also rejects human nature realism and, like
Waltz, refers to himself as a supporter of structural realism (Mearsheimer 2001, 20). One of the basic tenets of political realism (whether human nature realism or
structural realism) is, first, that the
second, that states
realists,

states are the main, if not the only, relevant actors

compete for power

in the international arena.

in the context of international relations;

and

Moral considerations in international affairs, according to

are secondary when set against the states primary goal, namely its own security and

survival . But while human nature realists such as Morgenthau explain the struggle for power as a result of human beings natural inclinations, structural realists like
Waltz and Mearsheimer argue that conflicts in the international arena do not stem from human nature, but from the very structure of the international system of states

it is this structure that compels individuals to act as they do


in the domain of international affairs. And one distinguishing feature of the international system of states is its anarchical
structure, i.e. the lack of a central government analogous to the central governments that exist in the context of domestic politics. It means that each
individual state is responsible for its own integrity and survival. In the absence of a superior
authority, over and above the power of each sovereign state, political leaders often feel compelled to favor security over
(Mearsheimer 2001, 18). According to Waltz and Mearsheimer,

morality , even if, all other things being considered, they would naturally be more inclined to trust and to cooperate with political leaders of other states. On the other
hand, when political leaders do trust and cooperate with other states, it is not necessarily their benevolent nature that motivates them to be cooperative and trustworthy, but,
again, it is the structure of the system of states that compels them. The concept of human nature, as we can see, does not play a decisive role here. Because Waltz and
Mearsheimer depart from human nature realism, their version of political realism has also sometimes been called neo-realism (Booth 1991, 533). Thus,

human beings turn out to become morally enhanced

in the future,

even if

humankind may still have to

face the same scary scenarios described by some moral enhancement theorists. This is likely to happen if, indeed,
human beings remain compelled to cooperate within the present structure of the system of states. Consider, for instance, the incident with
a Norwegian weather rocket in January 1995. Russian radars detected a missile that was initially suspected of being on its way to reach Moscow in five minutes. All levels of
Russian military defense were immediately put on alert for a possible imminent attack and massive retaliation. It is reported that for the first time in history a Russian president
had before him, ready to be used, the nuclear briefcase from which the permission to launch nuclear weapons is issued. And that happened when the Cold War was already
supposed to be over! In the event, it was realized that the rocket was leaving Russian territory and Boris Yeltsin did not have to enter the history books as the man who started

under the crushing pressure of having to decide in such a short


time, and on the basis of unreliable information, whether or not to retaliate, even a morally
enhanced Yeltsin might have given orders to launch a devastating nuclear response and that in
the third world war by mistake (Cirincione 2008, 382).3 But

spite of strong moral dispositions to the contrary.

Writing for The Guardian on the basis of recently declassified

documents, Rupert Myers reports further incidents similar to the one of 1995. He suggests that as more states strive to acquire nuclear capability, the danger of a major nuclear
accident is likely to increase (Myers 2014).

What has to be changed, therefore, is not human moral dispositions, but the

very structure of the political international system of states within which we currently live. As far as major
threats to the survival of humankind are concerned, moral enhancement might play an important role in the future only to the extent that it will help humankind to change the

moral enhancement may possibly have desirable results in some areas of human cooperation that do not badly threaten
our security such as donating food, medicine, and money to poorer countries it will not motivate political leaders to dismantle
structure of the system of states. While

their nuclear weapons . Neither will it deter other political leaders from pursuing nuclear
capability, at any rate not as long as the structure of international politics compels them to see
prospective cooperators in the present as possible enemies in the future. The idea of a structure
should not be understood here in metaphysical terms, as though it mysteriously existed in a transcendent world and had the magical power of determining leaders decisions in

in the absence of the


kind of security that law-enforcing institutions have the force to create , political leaders will often
fail to cooperate, and occasionally engage in conflicts and wars, in those areas that are critical to their security
this world. The word structure denotes merely a political arrangement in which there are no powerful law-enforcing institutions. And

and

survival. Given the structure of international politics and the basic goal of survival, this is likely to continue to happen, even if, in

the future, political

leaders become less egoistic and power-seeking

through moral enhancement. On the other hand, since the

structure of the international system of states is itself another human institution, there is no reason to suppose that it cannot ever be changed. If people become morally
enhanced in the future they may possibly feel more strongly motivated to change the structure of the system of states, or perhaps even feel inclined to abolish it altogether. In my

addressing major threats to the survival of humankind in the future by means of


bioengineering is unlikely to yield the expected results , so long as moral enhancement is

view, however,

pursued within the present framework of the international system of states.

Realists hate war- realism solves conflict


Edelstein 10 (2010, David, PhD in Political Science, Associate Professor in the
Edmund A. Walsh School of Foreign Service and the Department of Government at
Georgetown University, Why realists dont go for bombs and bullets,
http://foreignpolicy.com/2010/07/21/why-realists-dont-go-for-bombs-and-bullets/)

Thanks to Steve Walt for inviting me to contribute to his blog while he is away on vacation. I have been a regular reader of Steves blog since it launched, and for my first post, I wanted
to pick up on a motif that I have seen running through Steves posts:
States

Will realists ever again support the use of military force by the United

Followers of this blog will by now have little doubt about how Walt felt about the Iraq War or how he views the prospects for U.S. success in Afghanistan. In fact, throughout the history of
his blog, I can only recall one case in which Walt advocated the use of U.S. military force (and I think the realist credentials in that case are rather dubious).

There is a common perception in the field of political science that realists are warmongering Neanderthals anxious to use military force at the drop of a hat. Attend any meeting (if you must) of the American Political Science
Association or the International Studies Association, and one will find realists derided as the "bombs and bullets
guys" as if we were all direct descendants of Curtis LeMay. What is notable about this and what has been notable about Steves blog is just
how infrequently realists have supported the use of American military
force . Take the U.S. interventions of the post-Cold War period: Panama, the Gulf War, Somalia, Haiti, Bosnia,
Kosovo, Afghanistan, and Iraq. Of those interventions, Afghanistan was the only one
that received anything close to strong support from most realists. Others, most
notably the Iraq War, received vehement opposition from the vast majority
of realists . Even in the case of Afghanistan, realists expressed trepidation about
the prospects for ultimate success despite early victories.
Go back to the Cold War, and realists like Kenneth Waltz and Hans Morgenthau were famously
opposed to the U.S. intervention in Vietnam . Lest one think this is an academic
phenomenon, realist policymakers like Brent Scowcroft were equally critical of the Bush
administrations actions in Iraq, and George F. Kennan was skeptical of the U.S. interventions in both Korea and
Vietnam. Today, should anyone dare to suggest the use of military force in new
contexts such as Iran, they are summarily dismissed by prominent realists . Not a
single (self-proclaimed or attributed) realist I know of has advocated the use of military force
against Iran in response to its apparent development of nuclear weapons, and most are adamantly opposed to it.

From one perspective, this opposition is surprising. It is realists, after all, who so value material power, in particular military capabilities. It is not difficult to understand why so many
would assume that realists are anxious to use military force because realists are anxious to focus on military capabilities as a primary explanatory variable for international politics.

But

it is precisely because realists have spent so much time studying

military force that they are also so reluctant to use military force . Though realists
themselves are divided on the question, many have concluded that the use of military force is often counterproductive,
inviting balancing coalitions that simply make life more difficult. Moreover, as I have argued elsewhere, using military force to reorder societies is very difficult and unlikely to succeed
except in uncommon circumstances.

Neorealism is the opposite of rational subjectivity the microeconomics-realism


analogy is false.
Bessner and Guilhot 15 (Fall 2015, Daniel, Assistant Professor in the Henry M. Jackson
School of International Studies at the University of Washington and a postdoctoral fellow in U.S.
foreign policy and international security at the John Sloan Dickey Center for International
Understanding at Dartmouth College, and Nicolas, Research Professor at the National Center
for Scientific Research in France and a visiting scholar at the Center for International Research
in the Humanities and Social Sciences at New York University, How Realism Waltzed Off:
Liberalism and Decisionmaking in Kenneth Waltz's Neorealism, International Security, Vol.
40, No. 2, Pages 87-118)

The relationship between neorealism and the question of decisionism becomes even more obvious when one
looks at the methodological choices that distinguished Theory of International Politics . Scholars
commonly assume that Waltz modeled neorealism after microeconomics .91 He had majored in
economics at Oberlin College before turning to political theory, and he often used economic analogies to explain his theory. Yet the emphasis on economics provides little
guidance in understanding the epistemological foundations of neorealism . Waltz may well have looked
occasionally to economics to establish a theory of politics, but, whether he knew so or not, he was observing a discipline whose epistemic status had been transformed by cybernetic approaches.92 All his
considerations about the nature and functions of a theory, about its relation to reality, and about its uses, point in another direction:

toward the field of cybernetics and

general system theory .93


The intellectual sources of neorealism appear both in Theory of International Politics and in its embryonic version published in the 1975 Handbook of Political Science.94 The major sources include Ludwig von
Bertalanffy's General System Theory (1968), Ross Ashby's Design for a Brain (1952) and his Introduction to Cybernetics (1956), Norbert Wiener's Cybernetics (1948), and David Easton's systemic functionalism in
political science, each of which was foundational to the development of cybernetics and system theory in general.95
Situating Theory of International Politics in the context of the cybernetic movement enables one to understand Waltz's work in relation to both rationalist theories of decisionmaking and to the decisionist

Theory of International Politics belonged to a growing body of political


science scholarship that applied cybernetic insights (when not simply cybernetic terminology) to political problems. Earlier
undertones of classical realism. It suggests that

examples in the field of international relations include Morton Kaplan's System and Process in International Relations (1957), Graham Allison's Essence of Decision (1971), and John Steinbruner's The Cybernetic
Theory of Decision (1974).96 Theory of International Politics was in fact a late addition to this broader intellectual movement.
These works sought to make international relations more scientific while addressing immediate political issues, particularly policy formulation and decisionmaking. In the 1960s and 1970s,

cybernetics and system theory appealed to political scientists because they offered an
alternative to rationalistic theories of decisionmaking and seemed better adapted to the
complexity presented for policy research. The most important problem international relations theorists

unprecedented growth of governmental bureaucracies and to the challenge organizational

confronted was that, in the new institutional era, political decisions were largely impersonal outcomes of exceedingly complex procedures that traditional models of decisionmaking, which implied a centralized
decisionmaker, a set of transitive preferences, and a linear decision process, were unable to explain.

the most appealing characteristic of cybernetics and system theory for Waltz was that they
moved away from notions of decision and choice . Indeed, they were explicitly conceived as

Perhaps

an alternative to rational choice models and operated from a theoretical base

fundamentally different from rational theory. 97 They rejected formal decision theories,
and game theory in particular, as better suited to an ideal world where alternatives were known and the capacity to compute them (i.e., rationality) widely available. In
the real world, system theorists claimed, choices were made in a murkier fashion, and the number of actual possibilities was vastly superior to
the limited computing capacities of human actors , who were obviously not rational . Organizational and
bureaucratic processes were also part of the decisional machinery .98 In their own way, system theorists echoed the critique of
rational choice theories made by classical realists from the late 1940s onward, but in a new, formal, and non-ideological language.99

System theory, for its part, made it possible to explain the outcome of thoughtful, strategic decisions
without assuming any underlying deliberative thought process, rationality, or
intention . As Steinbruner declared, Much of the work [in cybernetics] has been directed precisely at the problem of
explaining highly successful behavior (usually called adaptive) without assuming elaborate decision-making
mechanisms.100 To do so, system theory and cybernetics shifted the emphasis from teleological, outcomedriven models to models that emphasized process and recipe-following , while being deprived of any
form of entelechy. Servomechanics, feedback loops, and other physical analogies were deployed to explain efficient adaptive behavior without ascribing any rationality to political actors or, indeed, taking much
account of actors.101 Before the rise of system theory, to quote one of Waltz's sources, notions of teleology and directiveness appeared to be outside the scope of science and to be the playground of mysterious,

Goal-seeking behavior, as Ross Ashby had explained,


was now understood on the basis of organizational patterns totally deprived of willful purpose or
self-determination.103 Self-steering could be represented as a process devoid of teleology and will. To put it differently, one could replicate, modelize, or simulate decisional processes
supernatural or anthropomorphic agencies.102 Cybernetics made it possible to replace decisions with mechanistic processes.

without assuming the existence of a central and omnipotent decisionmaker.

The removal of the decisionmaker was a crucial aspect of Theory of International Politics. System theory enabled Waltz to
describe international politics without recourse to a concept of decision. How decisions were made no longer mattered, because cybernetic models made it possible to describe their outcomes without implying
purpose, intention, or rationality. As Shklar clearly saw, system and decision were mutually exclusive ways of talking about the same thing. In the system approach, whether the concept of decision was
considered an old anthropomorphic notion tied to obscure forces, or an unrealistic assumption of rationality, it could be jettisoned in favor of anonymous adaptive processes. The new approach to international
politics associated today with neorealism was, in fact, already present in a number of cybernetic works.104

Theory of International Politics thus rested on methodological choices that excluded any
rationality assumption (i.e., the idea that humans were rational actors). Neorealist theory,
Waltz wrote in the book, requires no assumption of rationality or of constancy of will on the part of all the
actors.105 Foreign-policy making is such a complicated business, he later asserted, that one cannot
expect of political leaders the nicely calculated decisions that the word rationality suggests. 106 And
in one of his last interviews, Waltz confessed that he had no idea what the expression rational actor meant.107 Mearsheimer is therefore correct to suggest that Waltz's putative rationality assumption is a
perplexing case of collective self-delusion among international relations theorists, who have usually overlooked both the context in which Waltz was writing and the intellectual sources that informed Theory of
International Politics.108

One reason for the persistence of this disciplinary canard is that international relations scholars
have tended to assume that Waltz adopted classical microeconomic foundations for his theory , and
in particular the notion of a rational actor. Not quite: Waltz made it clear that his theory
of international politics was not modeled after the neoclassical economic model
assuming rational actors acting in perfectly competitive markets (given that such a model would end up with
harmonious equilibria requiring no policing, as some critics have noticed).109 To the extent that he drew inspiration from economic theories, his reference was the theory of oligopolistic markets.110

There was, however, a much more formidable obstacle that prevented Waltz from adopting the rational actor assumption, which did in fact come straight from
microeconomics: Kenneth Arrow's impossibility theorem . As a trained economist, Waltz could not ignore
that the rational actor assumption led to the impossibility of rational collective
choice unless that choice was undemocratic and coercive, as Arrow had shown.111 Even for the new economists of rational choice, the fact that a decision was
political made it de facto either irrational or authoritarian .112 As collectives, states were either irresolute or irrational if they were democracies,
or their foreign policy decisions had to be insulated from the democratic process in order to attain rationality. In other words, the rationality assumption would have brought Waltz back to where he did not want to

His explicit reluctance to accept the


notion that states could be rational points directly to the state of the discussion of rationality
after Arrow and to the ideological implications of the rationality assumption .113
benamely, in the chorus of classical realists calling for the reinstatement of preliberal, pre-democratic foreign policy practices.

2AC SECURITY SOLVES CHINA


Reforming U.S. regional posturing is key to sustainable securitization in China
different levels of cooperation are unique to prevent forceful posturing
Jennifer Lind 16 - Associate Professor of Government, Dartmouth College (Sustainable
Security: Rethinking American National Security Strategy, 2016,
http://tobinproject.org/sites/tobinproject.org/files/assets/Sustainable%20Security%20-%20All
%20Chapters.pdf) hk
Strategic Reassurance. As argued earlier,

a serious risk of the US alliance system in East Asia is that the US

regional force posture necessary to uphold the credibility of US security guarantees is


deeply threatening to China. Because of the cost, instability, and souring of bilateral relations that its East Asian forward presence is
causing in US-China relations, Washington should look for ways to reassure Beijing . To be sure, perhaps it is
ultimately not possible to square the circle of maintaining a force posture in East Asia that would both reassure American allies and
not threaten Beijing. But

Washington should only arrive at this conclusion after careful

study . US officials and military analysts should discuss: are there aspects of American force
structure or operations that could be reformed so to not overly antagonize China ? Scholars have
already begun a productive discussion about strategic reassurance; James Steinberg and Michael OHanlon identify four tools
(restraint, reciprocity, transparency, and resilience), arguing it is crucial to find ways of transcending or minimizing such a classic
security dilemma.137 Washington should work with Beijing in non-military areas to build cooperation and trust. Lyle Goldstein
advocates a spiral of cooperation, noting that, Since the Sunnylands summit, a string of positive, albeit small-scale joint activities
has been undertaken by the US and Chinese armed forces. Goldstein points to encouraging

progress in the areas of


carbon emissions, trade, easing visa requirements, and military confidence-building. 138 Although
Washington is unlikely to be so fearful of a US-China spiral such that it would end American alliances in the region, American
military leaders and foreign policy officials should be looking for ways in which they can adapt
US policy so to avoid creating undue fear in Beijing. In sum, in this era of strategic and budgetary change, in
which Americans continue to support the countrys Asian alliances, foreign policy leaders should
fix these alliances in ways that minimize their costs and risks, and better serve the US strategic
interest.

Effective national strategies are key to a sustainable way of securitization no ev


specific to this
Jeremi Suri & Benjamin Valentino 16 Jeremi is a Distinguished Professor for Global
Leadership, History, and Public Policy, University of Texas at Austin and Benjamin is an
Associate Professor of Government, Dartmouth College (Sustainable Security: Rethinking
American National Security Strategy, 2016,
http://tobinproject.org/sites/tobinproject.org/files/assets/Sustainable%20Security%20-%20All
%20Chapters.pdf) hk
American national interests are best served when short-term foreign policy decisions are woven
into a strategic fabric that endures. Effective national strategies assign priorities to guide
behavior in crises, they allocate resources for worthwhile investments, they affirm valuable relationships, and they caution
against temptations and traps. Effective national strategies also educate the public , they create

clear expectations, and they provide criteria for assessing success and failure. Strategies tell us
clearly what is worth fighting for and what is not. All together, these qualities contribute to what we
call a sustainable national strategy. We are concerned with the formulation and implementation of a new American
national strategy that can serve the nations interests at reasonable cost, with consistent support, and continuing benefits to
Americans and non-Americans alike. A

more sustainable strategy, like the Open Door in the late nineteenth century and
containment in the Cold War, must be simple, resilient, and compatible with American institutions and
values.

2AC SECURITY SOLVES WARMING


Perm do both solves contextualized approaches of securitization in the context of climate
change are key
Jonna Nyman & Jinghan Zeng 16 Jonna: Department of Politics and International
Relations, University of
Leicester and Jinghan: Department of Politics and International Relations, Royal Holloway,
University of London (Securitization in Chinese climate and energy politics, 24 JAN 2016
online, WIREs Clim Change, 7: 301313. Wiley online library) hk
In real terms, Chinas

energy consumptionin particular its coal dependenceposes an increasing threat to


global environmental stability as well as the countrys own ecological securit y. China is still developing
its approaches to energy and climate change, and is also increasingly linking the two issues together. The dynamics of
securitization differ, and there is growing focus on nontraditional security issues. This also suggests that securitization may
not be best avoided. Clearly, a contextualized approach is needed to study securitization
in practice , in particular when studying securitization in a non-Western context .
Ultimately, climate change will pose challenges to Chinas securitynational security,
environmental security, and energy security, alongside human security and water security.
Securitization is one potential way to approach it. But energy and climate security also have to be balanced alongside
other important security concerns, including economic security and development, so there is no simple solution here. One of the
biggest challenges remains Chinas weak energy governance, which makes policy implementation difficult. Use of renewable energy
in China is increasing, but significant obstacles remain.85 One alternative is to consider energy conservation as a security issue86
after all, it would ease both energy security and climate change .

Energy security and climate change are two of the


most serious issues facing Chinas continued development. Its policy choices play a key role in shaping global
climate and energy dynamics. Thus, we argue that the growing importance of security in Chinas energy and
climate debates is an important area for further research, alongside the implications of
securitization in these cases.

2AC UTIL
Ethical obligations are tautologicalthe only coherent rubric is to maximize
number of lives saved
Greene 2010 Associate Professor of the Social Sciences Department of Psychology Harvard
University (Joshua, Moral Psychology: Historical and Contemporary Readings, The Secret Joke
of Kants Soul, www.fed.cuhk.edu.hk/~lchang/material/Evolutionary/Developmental/GreeneKantSoul.pdf, WEA)

What turn-of-the-millennium science is telling us is that human moral judgment is not


a pristine rational enterprise, that our moral judgments are driven by a hodgepodge of emotional
dispositions, which themselves were shaped by a hodgepodge of evolutionary forces, both
biological and cultural. Because of this, it is exceedingly unlikely that there is
any rationally coherent normative moral theory that can
accommodate our moral intuitions . Moreover, anyone who claims to have such a theory, or even
part of one,

almost certainly doesn't . Instead, what that person probably has is a moral rationalization.

It seems then, that we have somehow crossed the infamous "is"-"ought" divide. How did this happen? Didn't Hume (Hume, 1978) and Moore (Moore,
1966) warn us against trying to derive an "ought" from and "is?" How did we go from descriptive scientific theories concerning moral psychology to
skepticism about a whole class of normative moral theories? The answer is that we did not, as Hume and Moore anticipated, attempt to derive an
"ought" from and "is." That is, our method has been inductive rather than deductive. We have inferred on the basis of the available evidence that the
phenomenon of rationalist deontological philosophy is best explained as a rationalization of evolved emotional intuition (Harman, 1977).

Missing the Deontological Point


I suspect that rationalist deontologists will remain unmoved by the arguments presented here. Instead, I
suspect, they will insist that I have simply misunderstood what Kant and likedeontologists are all about . Deontology, they will say, isn't about this intuition or that
intuition. It's not defined by its normative differences with consequentialism. Rather, deontology is about taking
humanity seriously. Above all else, it's about respect for persons. It's about treating others as fellow rational creatures rather than as mere
objects, about acting for reasons rational beings can share. And so on (Korsgaard, 1996a; Korsgaard, 1996b). This is, no doubt, how
many deontologists see deontology. But this insider's view, as I've suggested, may be
minded

misleading . The problem, more specifically, is that it defines deontology in terms of values
that are not distinctively deontological , though they may appear to be from the inside. Consider the following
analogy with religion. When one asks a religious person to explain the essence of his religion,
one often gets an answer like this: "It's about love, really. It's about looking out for other people, looking beyond oneself.
It's about community, being part of something larger than oneself." This sort of answer accurately captures the
phenomenology of many people's religion, but it's nevertheless inadequate for distinguishing
religion from other things. This is because many, if not most, non-religious people aspire to love deeply, look out for other people, avoid
self-absorption, have a sense of a community, and be connected to things larger than themselves. In other words, secular humanists and atheists can
assent to most of what many religious people think religion is all about. From a secular humanist's point of view, in contrast, what's distinctive about
religion is its commitment to the existence of supernatural entities as well as formal religious institutions and doctrines. And they're right. These things
really do distinguish religious from non-religious practices, though they may appear to be secondary to many people operating from within a religious
point of view.

In the same way, I believe that most of the

standard deontological/Kantian self-characterizatons fail to

distinguish deontology from other approaches to ethics . (See also Kagan (Kagan, 1997, pp. 70-78.) on the
difficulty of defining deontology.) It seems to me that

consequentialists, as much as anyone else, have respect for

persons , are against treating people as mere objects, wish to act for reasons that
rational creatures can share , etc. A consequentialist respects other persons, and refrains
from treating them as mere objects, by counting every person's well-being in the decisionmaking process . Likewise, a consequentialist attempts to act according to reasons that
rational creatures can share by acting according to principles that give equal weight to
everyone's interests , i.e. that are impartial. This is not to say that consequentialists and deontologists don't differ. They
do. It's just that the real differences may not be what deontologists often take them to be.
What, then, distinguishes deontology from other kinds of moral thought? A good strategy for answering this question is to start with concrete
disagreements between deontologists and others (such as consequentialists) and then work backward in search of deeper principles. This is what I've

If you ask a
deontologically-minded person why it's wrong to push someone in front of speeding trolley in
order to save five others, you will getcharacteristically deontological answers. Some will
attempted to do with the trolley and footbridge cases, and other instances in which deontologists and consequentialists disagree.

be tautological: "Because it's murder!" Others will be more sophisticated: "The ends don't
justify the means." "You have to respect people's rights." But , as we know, these answers don't really explain
anything, because if you give the same people (on different occasions) the trolley case or the loop case (See
above), they'll make the opposite judgment , even though their initial explanation concerning the footbridge case applies
equally well to one or both of these cases. Talk about rights, respect for persons, and reasons we can share are
natural attempts to explain, in "cognitive" terms, what we feel when we find ourselves having
emotionally driven intuitions that are odds with the cold calculus of consequentialism . Although these
explanations are inevitably incomplete, there seems to be "something deeply right" about
them because they give voice to powerful moral emotions . But, as with many religious
people's accounts of what's essential to religion, they don't really explain what's distinctive
about the philosophy in question.

Prefer utilitarian calculuses in the context of security its a question of survival


Peter OBoyle 16 - University of British Columbia (Rethinking securitization: water scarcity
and energy security in China, April 2016,
https://open.library.ubc.ca/cIRcle/collections/ubctheses/24/items/1.0300163) hk
Water resource availability affects energy security in China because energy from coal is crucial for Chinas
energy security, and water is key for energy production from coal. The next section undertakes a systematic inquiry into this topic,
using the Copenhagen Schools securitization framework and its development as proposed by Mely Caballero-Anthony and Ralf
Emmers, as well as some new additions proposed to improve securitization analysis frameworks. Buzan, Weaver and de Wilde stress

security is about survival , arguing that something becomes an international security issue
when it is presented as posing an existential threat to a referent object , such that the special
nature of the threat justifies the use of extraordinary [or emergency] measures to deal with it .
that

They argue that to securitize an issue 48 takes politics beyond the established rules of the game and frames the issue as either as a
special kind of politics or as above politics. Thus, 49 if one can argue that something overflows the normal political logic of weighing
issues against each other this must be the case because it can upset the entire process of weighing as such: If

we do not
tackle this problem, everything else will be irrelevant (because we will not be here or will

not be free to deal with it in our own way ).50 As such security is a self-referential
practice, because it is in the practice that the issue becomes a security issue - not necessarily because a
real existential threat exists but because the issue is presented as such a threat. They advocate three key criteria for securitization
analysis: 51 (i) referent objects - things

that are seen to be existentially threatened and that have a


legitimate claim to survival ; (2) securitizing actors, who securitize an issue by articulating the existence 52 of
threat(s) to the survival of specific referent objects ; (3) the completion of the process of 53 securitization,
requiring a speech act by securitizing actors which convinces a specific audience of the
existential nature of the threat.

AT: ENDLESS WAR IMPACT


No risk of endless warfare
Gray 7Director of the Centre for Strategic Studies and Professor of International Relations and Strategic Studies at the University of Reading,
graduate of the Universities of Manchester and Oxford, Founder and Senior Associate to the National Institute for Public Policy, formerly with the
International Institute for Strategic Studies and the Hudson Institute (Colin, July, The Implications of Preemptive and Preventive War Doctrines: A
Reconsideration, http://www.ciaonet.org/wps/ssi10561/ssi10561.pdf)
7. A policy that favors preventive warfare expresses a futile quest for absolute security. It could do so. Most controversial policies contain within them the possibility of

In the hands of a paranoid or boundlessly ambitious political leader, prevention could be a policy for
endless warfare. However, the American political system, with its checks and balances, was
designed explicitly for the purpose of constraining the executive from excessive folly. Both the
Vietnam and the contemporary Iraqi experiences reveal clearly that although the conduct of war is an
executive prerogative, in practice that authority is disciplined by public attitudes. Clausewitz made this point superbly with his designation of the
misuse.

passion, the sentiments, of the people as a vital component of his trinitarian theory of war. 51 It is true to claim that power can be, and indeed is often, abused, both personally
and nationally. It is possible that a state could acquire a taste for the apparent swift decisiveness of preventive warfare and overuse the option. One might argue that the easy
success achieved against Taliban Afghanistan in 2001, provided fuel for the urge to seek a similarly rapid success against Saddam Husseins Iraq. In other words, the delights of
military success can be habit forming. On balance, claim seven is not persuasive, though it certainly contains a germ of truth. A country with unmatched wealth and power,
unused to physical insecurity at homenotwithstanding 42 years of nuclear danger, and a high level of gun crimeis vulnerable to demands for policies that supposedly can

we ought not to endorse the argument that the United States should eschew the preventive
war option because it could lead to a futile, endless search for absolute security. One might as
well argue that the United States should adopt a defense policy and develop capabilities shaped strictly for homeland security approached in a narrowly geographical
sense. Since a president might misuse a military instrument that had a global reach, why not deny
the White House even the possibility of such misuse? In other words, constrain policy ends by
limiting policys military means. This argument has circulated for many decades and, it must be admitted, it does have a certain elementary logic. It is
the opinion of this enquiry, however, that the claim that a policy which includes the preventive option might lead to
a search for total security is not at all convincing. Of course, folly in high places is always possible, which is one of the many
reasons why popular democracy is the superior form of government . It would be absurd to permit the fear of a futile and
dangerous quest for absolute security to preclude prevention as a policy option. Despite its
absurdity, this rhetorical charge against prevention is a stock favorite among preventions
critics. It should be recognized and dismissed for what it is, a debating point with little
pragmatic merit. And strategy, though not always policy, must be nothing if not pragmatic.
restore security. But

Impossible to mobilize support for lashout


Mandelbaum 11 (Michael Mandelbaum, A. Herter Professor of American Foreign Policy, the
Paul H. Nitze School of Advanced International Studies, Johns Hopkins University, Washington
DC; and Director, Project on East-West Relations, Council on Foreign Relations, CFR 90th
Anniversary Series on Renewing America: American Power and Profligacy, Jan 2011)
http://www.cfr.org/publication/23828/cfr_90th_anniversary_series_on_renewing_america.h
tml?cid=rss-fullfeed-cfr_90th_anniversary_series_on011811&utm_source=feedburner&utm_medium=feed&utm_campaign=Feed:+cfr_main+
(CFR.org+-+Main+Site+Feed
HAASS: Michael, I think I know the answer to this question, but let me ask you anyhow, which is, the last 10 years of American foreign policy has been
dominated by two extremely expensive interventions, one in Iraq, one now in Afghanistan. Will this sort of pressure both accelerate the end,
particularly of Afghanistan? But, more important, will this now -- is

this the end of that phase of what we might call "discretionary

American interventions?" Is this basically over? MANDELBAUM: Let's call them wars of choice. (Laughter.) HAASS: I was trying to be
uncharacteristically self-effacing here. But clearly it didn't hold. Okay. MANDELBAUM: I think it is , Richard. And I think that this period
really goes back two decades. I think the wars or the interventions in Somalia, in Bosnia, in Kosovo, in Haiti belong with the
interventions in Afghanistan and Iraq, although they were undertaken by different administrations for different reasons, and had different costs. But all

of them ended

up in the protracted, unexpected, unwanted and expensive task of nation building.


Nation building has never been popular. The country has never liked it. It likes it even less now.
And I think we're not going to do it again. We're not going to do it because there won't be enough money. We're not
going to do it because there will be other demands on the public purse. We won't do it because we'll be busy enough doing
the things that I think ought to be done in foreign policy. And we won't do it because it will be clear to politicians that the
range of legitimate choices that they have in foreign policy will have narrowed and will
exclude interventions of that kind. So I believe and I say in the book that the last -- the first two post-Cold War decades can
be seen as a single unit. And that unit has come to an end.

No risk of lash-out institutional safeguards check


Allen Buchanan 7, Professor of Philosophy and Public Policy at Duke, 2007 (Preemption:
military action and moral justification, pg. 128)
the higher the stakes in acting and in particular
the greater the moral risk, the higher are the epistemic requirements for justified action. The
decision to go to war is generally a high stakes decision par excellence and the moral risks are especially great, for two reasons. First, unless one is justified in
The intuitively plausible idea behind the 'irresponsible act' argument is that, other things being equal,

going to war, one's deliberate killing of enemy combatants will he murder, indeed mass murder. Secondly, at least in large-scale modem war, it is a virtual certainty that one will kill innocent people even if one is
justified in going to war and conducts the war in such a way as to try to minimize harm to innocents. Given these grave moral risks of going to war, quite apart from often substantial prudential concerns, some
types of justifications for going to war may simply be too subject to abuse and error to make it justifiable to invoke them. The 'irresponsible act' objection is not a consequentialist objection in any interesting
sense. It does not depend upon the assumption that every particular act of going to war preventively has unacceptably bad consequences (whether in itself or by virtue of contributing lo the general acceptance of a
principle allowing preventive war); nor does it assume that it is always wrong lo rely on a justification which, if generally accepted, would produce unacceptable consequences. Instead, the "irresponsible act'

The 'irresponsible act'


objection to preventive war is highly plausible if but only ifone assumes that the agents who
would invoke the preventive-war justification are, as it were, on their own in making the decision to go to war preventively.
objection is more accurately described as an agent-centered argument and more particularly an argument from moral epistemic responsibility.

In other words, the objection is incomplete unless the context of decision-making is further specified. Whether the special risks of relying on the preventive-war justification are unacceptably high will depend,
inter alia, upon whether

the decision-making process includes effective provisions for reducing those special

risks. Because the special risks are at least in significant part epistemicdue to the inherently speculative character of the preventive war-justificationthe epistemic context of the decision is crucial. Because
institutions can improve the epistemic performance of agents, it is critical to know what the institutional context of the preventive-war
decision is, before we can regard the 'irresponsible agent' objection as conclusive. Like the 'bad practice' argument, this second objection to preventive war is inconclusive because it does not consider and rule

well-designed institutions for decision-making could address the problems that


would otherwise make it irresponsible for a leader to invoke the preventive-war justification.
outthe possibility that

AT: SERIAL POLICY FAILURE


Our theory doesnt cause a self-fulfilling prophecy states only balance to a limit
Rosato and Schuessler 11 Assistant Professor of Political Science at the University of Notre Dame AND Assistant Professor of
Strategy and International Security at the Air War College
(Sebastian and John, A Realist Foreign Policy for the United States, Perspectives on Politics December 2011 Vol. 9 No. 4, dml)

Given the dangers of weakness, great

powers should balance against more powerful peer


competitors. What does this entail? Most important, it means that they should build up their military and
economic resources. The reason is that by doing so they will array more capabilities against a competitor,
thereby increasing the probability that they will deter it from attacking them or that they
will defeat it in the event that deterrence fails. As Charles Glaser explains, All else being equal, a state is more secure
when it possesses the military capabilities required to protect its territory from attack. These
capabilities could include those required to deter attack . . . They could also include those required to directly protect the state. 36
Balancing also has a crucial signaling component to it. The balancer must not only build up its capabilities, but must also make it
clear to the more powerful state that it will vigorously oppose any attempt at expansion. In effect, explains Mearsheimer, the

balancer draws a line in the sand and warns the aggressor not to cross it. 37 Aspiring
balancers can also achieve their aims by allying with other states against an especially powerful peer competitor. Kenneth Waltz
offers a neat summary of the logic, noting that states form alliances in the hope of achiev[ing] enough defensive or deterrent
strength to dissuade adversaries from attacking. 38 Mearsheimer makes essentially the same point: recruiting allies increases the
amount of repower confronting an aggressor, which in turn increases the likelihood that deterrence will work. 39 As in the case of
building up ones own capabilities, alliances

are more likely to have their desired deterrent effect if the


allies make it clear to their common adversary that they are resolved to oppose it in the event
that it attempts to expand. 40 Our recommendation that great powers should balance does not imply that they
should build up their capabilities without limit. Taken to its logical extreme, this would be a prescription
for bankruptcy. Therefore, states should increase their power until they have the resources to confront
their opponent with the prospect of having to ght a protracted war of attrition , a prospect that should
be sufcient to deter most aggressors. 41 Allies should help in this cause, as even highly expansionist states will be reluctant to court
war on multiple fronts. Finally, in the nuclear age, a secure second-strike capability is essential to deter nuclear-armed adversaries.
42

Self-fulfilling prophecy goes aff maximizes vulnerability


Rosato and Schuessler 11 Assistant Professor of Political Science at the University of Notre Dame AND Assistant Professor of
Strategy and International Security at the Air War College
(Sebastian and John, A Realist Foreign Policy for the United States, Perspectives on Politics December 2011 Vol. 9 No. 4, dml)

Another potential objection comes from the defensive variant of realism. The basic argument

is that balancing
aggressively against a powerful adversary may make it feel less secure, thereby increasing the
likelihood that it attacks. Put slightly differently, balancing can generate a spiral of fear that ultimately leads an adversary
to lash out at those balancing against it. 44 This analysis yields an obvious alternative prescription: rather than balancing
and triggering a spiral of fear, a state should try to reassure its adversary by not balancing
especially aggressively. The effect of this strategy, in many cases, is that the state trying to do the

reassuring has now increased its vulnerability. This is a reasonable course of action as long as the balancer is
quite condent that its potential adversary has benign intentions. And, indeed, defensive realists argue that, under a range of
conditions, states

can achieve such condence. 45 Our position, however, is that intentions are unknowable,
and even if known, are subject to change. 46 This being the case, no state should knowingly put itself in a position
of weakness.

AT: STUCTURAL VIOLENCE

Turnthe case is a gateway to the alt


Folk 78 Peace Studies Professor, Bethany College (Jerry, Peace Education-Peace Studies Programs, Peace Change 5.1, AG)

Those proponents of the positive peace approach who reject out of hand the work of researchers and educators coming to the field
from the perspective of negative peace too easily forget that the

prevention of a nuclear confrontation of global


the prerequisite for all other peace research, education, and action. Unless such a
confrontation can be avoided there will be no world left in which to build positive peace. Moreover, the
dimensions is

blanket condemnation of all such negative peace oriented research, education or action as a reactionary attempt to support and
reinforce the status quo is doctrinaire. Conflict theory and resolution, disarmament studies, studies of the international system and
of international organizations, and integration studies are in themselves neutral. They do not intrinsically support either the status
quo or revolutionary efforts to change or overthrow it. Rather they offer a body of knowledge which can be used for either purpose or
for some purpose in between. It

is much more logical for those who understand peace as positive peace to
integrate this knowledge into their own framework and to utilize it in achieving their own purposes. A balanced
peace studies program should therefore offer the student exposure to the questions and concerns which occupy those who view the
field essentially from the point of view of negative peace.

Nuke war threat is real and o/w structural and invisible violence---their expansion
of structural violence to an all-pervasive omnipresence makes preventing war
impossible
Boulding 1977 Kenneth, Prof Univ. of Michigan and UC Boulder, Journal of Peace
Research; 14; 75 p. Boulding p. 83-4

Finally, we come to the great Galtung metaphors of structural

violence and positive peace. They are metaphors


rather than models, and for that very reason are suspect. Metaphors always imply models and
metaphors have much more persuasive power than models do, for models tend to be the
preserve of the specialist. But when a metaphor implies a bad model it can be very dangerous,
for it is both persuasive and wrong. The metaphor of structural violence I would argue falls right
into this category. The metaphor is that poverty, deprivation, ill health, low expectations of life, a condition in which more than half the
human race lives, is like a thug beating up the victim and taking his money away from him in the street, -or it is like a conqueror stealing the land of
the people and reducing them to slavery. The

implication is that poverty and its associated ills are the fault of
the thug or the conqueror and the solution is to do away with thugs and conquerors. While there
is some truth in the metaphor, in the modem world at least there is not very much. Violence,
whether of the streets and the home, or of the guerilla, of the police, or of the armed forces , is a
very different phenomenon from poverty. The processes which create and sustain poverty are
not at all like the processes which create and sustain violence, although like everything else in
the world, everything is somewhat related to everything else. There is a very real problem of the
structures which lead to violence, but unfortunately Galtungs metaphor of structural violence as

he has used it has diverted attention from this problem. Violence in the behavioral sense, that is,
somebody actually doing damage to somebody else and trying to make them worse off, is a threshold phenomenon , rather like the
boiling over of a pot. The temperature under a pot can rise for a long time without its boiling
over, but at some threshold boiling over will take place. The study of the structures which underlie violence are a
very important and much neglected part of peace research and indeed of social science in
general. Threshold phenomena like violence are difficult to study because they represent breaks
in the system rather than uniformities. Violence, whether between persons or organizations,
occurs when the strain on a system is too great for its ~s~trength. The metaphor here is that
violence is like what happens when we break a piece of chalk. Strength and strain, however,
especially in social systems, are so interwoven historically that it is very difficult to separate
them. The diminution of violence involves two possible strategies, or a mixture of the two; one is
the increase in the strength of the system, ~the other is the diminution of the strain. The
strength of systems involves habit, culture, taboos, and sanctions, all these things, which enable
a system to stand Increasing strain without breaking down into violence. The strains on the
system are largely dynamic in character, such as arms races, mutually stimulated hostility,
changes in relative economic position or political power, which are often hard to identify . Conflict of
interest are only part of the strain on a system, and not always the most important part. It is very hard for people to know their interests, and
misperceptions of interests take place mainly through the dynamic processes, not through the structural ones. It

is only perceptions of
interest which affect peoples behavior, not the real interests, whatever these may be, and the
gap between perception and reality can be very large and resistant to change. However, what
Galitung calls structural violence (which has been defined by one unkind commentator as
anything that Galltung doesn~t like) was originally defined as any unnecessarily low
expectation of life, an that assumption that anybody who dies before the allotted span has been
killed, however unintentionally and unknowingly, by somebody else. The concept has been
expanded to include all the problems off poverty, destitution, deprivation, and misery. These are
enormously real and are a very high priority for research and action, but they belong to systems
which are only peripherally related to the structures which, produce violence. This is not to say
that the cultures of violence and the cultures of poverty are not sometimes related , though not all poverty
cultures are culture of violence, and certainly not all cultures of violence are poverty cultures. But the dynamics of poverty and the
success or failure to rise out off it are of a complexity far beyond anything which the metaphor
of structural violence can offer. While the metaphor of structural violence performed a service
in calling attention to a problem, it may have done a disservice in preventing us from finding the
answer.

Negative peace key to positive peace


Ferrer 6 Center for Integrative and Development Studies (Miriam, Costly Wars, Elusive Peace,
http://www.up.edu.ph/upforum.php?i=54&archive=yes&yr=2006&mn=3, AG)

Development specially equitable and sustainable development creates favorable conditions to achieve peace. But development

Conflagration results in the destruction


population, efforts and resources. Development, including sustainable development,
requires continuity and security of capital and human resource. For these reasons, war is inconsistent with or countercannot be sustained in a situation of persistent and intermittent conflict.
and/or dispersal of

productive to development objectives. Moreover, the military option of clobbering insurgencies has tended to exacerbate the
situation, creating a most common phenomenon described in peace studies as the spiral of violence. Military offensives come with

the militarization of communities, given that most domestic armed challenges are in the nature of guerrilla warfare. The resultant
human rights violations and injustices further drive people to violent action, which in turn invite greater military repression. A war
of attrition ensues, and the origin of the conflict is blurred by acts of reprisals by all sides. The conflict assumes a prolonged and
devastating character, giving rise to the high costs of war discussed. Negative

peace thus becomes necessary to provide a

favorable context for development and more thorough social transformation. Simply put, negative pertains to the
absence of war and other direct violence or actual hostilities. Negative peace creates the conditions for positive peace,
defined basically as the absence of structural violence and the presence of social justice; they complement each other.
Moreover, negative peace creates a break to the patterns that support war and earn war benefits for those
who exploit the conditions of war. Not surprisingly, there is much resistance to negative peace from the groups who benefit most
from its absence. Policies should therefore move from those that spin the spiral of violence to those that create a spiral of peace and
development. The term peace dividends reflects the positive potential of achieving negative peace. Expenditures

and
losses due to wars can be converted into peace dividends derived from savings of resources
otherwise used in war efforts and earnings otherwise lost because of the conflict. But beyond
monetary accounting are the renewed sense of well-being needed in building human security,
and the breaking of old patterns, relations and institutions to give way to new ones in the economic,
political and social spheres. Peace dividends include: Budgetary savings that translate into increased budget for social services
which will benefit the bigger society, including military men and their families. Indeed, military forces should be made aware that
although war benefits their corporate interests, as citizens, they will also shoulder the cost of war and suffer the same poor social
services and welfare benefits as the rest of society. Earnings from unhampered economic activities. Boosting of the environment
for non-war sustaining economic activities, favorable conditions for empowerment projects and the shift to production of socially
beneficial consumer goods and services. Return to a semblance of normalcy, lessened physical insecurity, and a greater sense of
well-being (being able to regain control of ones life). Disruption of old patterns and the reproduction of abusive power and
violence. Peace lessens the demand for war resources, to the disappointment of arms traders and suppliers. It removes the cover for
criminal activities like drugs smuggling and the black market for precious/semi-precious stones and timber. Creation of an
environment for other political, economic and social reforms, including those reforms demanded by insurgent groups. The Nexus
between Peace and Development This short article has tried to advance a

policy framework that sees the attainment


of negative peace as necessary to put in place positive peace and sustainable development. At the same time, it
proposes initial measures combining an end to open hostilities through political negotiations, with smallscale, immediate economic development packages as measures that can bring about negative peace. Through such
steps, the spiral of peace and development is unleashed. This perspective does away with the
unnecessary, even acrimonious, debate on whether peace precedes development or vice-versa. A
more dynamic, dialectal relationship between peace (positive and negative) and development (short-term
and sustainable) is established.

SPECIFIC DEFENSES

2AC APOCALYPTIC RHETORIC


Apocalyptic rhetoric motivates action on climate change it causes emancipation,
not climate fatigue
Beck 10 (Ulrich, Professor of Sociology at University of Munich, the British Journal of
Sociology Visiting Centennial Professor at the London School of Economics and Political
Sciences, and, since 2009, Senior Loeb Fellow at the Harvard Design School, Climate for
Change, or How to Create a Green Modernity?, Theory Culture Society 2010 27: 254)
Sixth thesis: The political explosiveness

of global risks is largely a function of their (re-)presentation in


the mass media. When staged in the media, global risks can become 'cosmopolitan events'. The
presentation and visualization of manufactured risk makes the invisible visible . It creates
simultaneity, shared involvement and shared suffering, and thereby creates the relevance for a global
public. Thus cosmopolitan events are highly mediatized, highly selective, highly variable, highly symbolic local and global, public
and private, material and communicative, reflexive experiences and blows of fate. To understand this, we have to draw upon the
picture of 'Mediapolis' so minutely and sensitively painted by Silverstone (2006) and the picture sketched much earlier by Dewey
(1946). There Dewey defends the thesis that it is not actions but their consequences which lie at the heart of politics. Although he
was not thinking of global warming, BSE or terrorist attacks, his theory can be applied perfectly to world risk society. A global public
discourse does not arise out of a consensus on decisions, but rather out of disagreement over the consequences of decisions.

Modern risk crises are constructed out of just such controversies over consequences. Although some
insist on seeing an overreaction to risk, risk conflicts do indeed have an enlightening function. They
destabilize the existing order but can also be seen as a vital step towards the construction
of new institutions . Global risk has the power to confuse the mechanisms of organized irresponsibility and
even to open them up for political action. This view of 'enforced enlightenment' and 'cosmopolitan realism'
opens up the possibility that the 'manufactured uncertainties' and 'manufactured insecurities' produced by world risk society
prompt transnational reflexivity, global cooperation, coordinated responses against the background of
'cosmopolitan communities of risk', so the same processes may also prompt much else besides. My emphasis on staging follows from
the fact that my central concept is not 'crisis' but 'new global risk'. Risks are, essentially, man-made, incalculable, uninsurable
threats and catastrophes which are anticipated but which often remain invisible and therefore depend on how they become defined
and contested in 'knowledge'. As a result their 'reality' can be dramatized or minimized, transformed or simply denied, according to
the norms which decide what is known and what is not. They are, to repeat myself, products of struggles and conflicts over
definitions within the context of specific relations of definitional power and the (in varying degrees successful) results of staging. If
this is the core understanding of risk, then this means that we

must attach major significance to media staging


and acknowledge the potential political explosiveness of the media. How does this correspond to empirical facts?
As Cottle (2009) argues, the release in early 2007 of the latest I nternational P anel on C limate C hange report
proved to be a transformative moment in the news career of climate change (IPCC, 2007). At first
climate change featured relatively infrequently in scientifically framed news reports , then it was
contested by a small group of news-privileged climate change sceptics, and finally it came of age as a widely
recognized 'global risk' demanding responses from all the world's nations. If IPCC predictions
and those of more recent scientific modelling come to pass over the next couple of decades, then
climate change may yet prove to be the most powerful of forces summoning a civilizational
community of fate into existence. The Western news media's spectacular visualization of climate
change, presenting dramatic and symbolic scenes collected from around the world, has undoubtedly
helped to establish the latter's status as a widely recognized global challenge and serves to illuminate a
third-generational modernity staged as global spectacle. Here the news media do not only function in terms of a global focusing of
events; rather, the news media adopt a more performative stand, actively enacting certain issues as 'global risks'. Images which
function in a more indexical sense to stand in for global processes of climate change now regularly feature across the news

landscape. And here some sections of the news media have sought to champion climate change awareness ,
often through visually arresting images which aim to register the full force and threat produced by global warming around the world.

In images such as these, the abstract science of climate change is rendered culturally
meaningful and politically consequential ; geographically remote spaces become literally
perceptible, 'knowable' places of possible concern and action. This performative use of visual
environmental rhetoric is not confined to selected newspapers; interestingly enough, it has
become mainstream. In this way the threat and reality of global climate change has been 'brought home', especially in the
West, as possibly 'the' global risk of the age. On the other hand, the continuing pull of the national within the world's news
formations and discourses cannot be underestimated. This is, of course, true in the case of wars. Wars continue to be reported
through spectacles tinted by national interests. However, as climate change moves into a new phase of national and international
contention, countries, corporations and citizens are also negotiating their respective roles and responsibilities, whether in respect of
national policies of mitigation and adoption, or through governmental support of developing countries confronting the worst effects
of global warming. Here, too, actions and reactions are often reported in and through national news prisms and frames of reference.
However, the

narrative of global risk is misinterpreted as a narrative of the Western 'emergency


imaginary' (Calhoun, 2004). It is not a 'singing into the apocalypse', and it is not simply a 'wake-up
call to reality'. Rather it is about expectation and anticipation, it is about a narrative to
dream differently . ' Emancipation ' is the key word. Either the ecological concern manages
to be at least as powerful as this hunger for modernization or it is condemned to repeated
failure.

2AC DETERRENCE
Infinite non-falsifiable root causes means root cause focus over specific
consequences dooms us to war
Moore 4 Professor of Law at the University of Virginia. He formerly served as the first
Chairman of the Board of the United States Institute of Peace and as the Counselor on
International Law to the Department of State. (John Norton, Winter, Beyond the Democratic
Peace: Solving the War Puzzle, 44 Va. J. Int'l L. 341, Lexis Law)

If major interstate war is predominantly a product of a synergy between a potential nondemocratic aggressor and an absence of
effective deterrence, what is the role of the many traditional "causes" of war? Past, and many

contemporary, theories of
war have focused on the role of specific disputes between nations, ethnic and religious
differences, arms races, poverty and social injustice, competition for resources, incidents and
accidents, greed, fear, perceptions of "honor," and many other factors. Such factors may well
play a role in motivating aggression or generating fear and manipulating public opinion. The reality, however,
is that while some of these factors may have more potential to contribute to war than others,
there may well be an infinite set of motivating factors, or human wants, motivating aggression. It is
not the independent existence of such motivating factors for war but rather the circumstances
permitting or encouraging high-risk decisions leading to war that is the key to more effectively
controlling armed conflict. And the same may also be true of democide. The early focus in the Rwanda
slaughter on "ethnic conflict," as though Hutus and Tutsis had begun to slaughter each other through spontaneous combustion,
distracted our attention from the reality that a nondemocratic Hutu regime had carefully planned and orchestrated a genocide
against Rwandan Tutsis as well as its Hutu opponents. n158 Certainly if we were able to press a button and end poverty, racism,
religious intolerance, injustice, and endless disputes, we would want to do so. Indeed, democratic governments must remain
committed to policies that will produce a better world by all measures of human progress. The broader achievement of democracy
and the rule of law will itself assist in this progress. No one, however, has yet been able to demonstrate the kind of robust correlation

given the difficulties in


overcoming many of these social problems, an approach to war exclusively dependent on their
solution may doom us to war for generations to come.
with any of these "traditional" causes of war that is reflected in the "democratic peace." Further,

[*394] A useful framework for thinking about the war puzzle is provided in the Kenneth Waltz classic Man, the State and
War, n159 first published in 1954 for the Institute of War and Peace Studies, in which he notes that previous thinkers about the causes
of war have tended to assign responsibility at one of the three levels of individual psychology, the nature of the state, or the nature of
the international system. This tripartite level of analysis has subsequently been widely copied in the study of international relations.
We might summarize my analysis in this classical construct by suggesting that the most critical variables are the second and third
levels, or "images," of analysis. Government structures, at the second level, seem to play a central role in levels of aggressiveness in
high-risk behavior leading to major war. In this, the "democratic peace" is an essential insight. The third level of analysis, the
international system, or totality of external incentives influencing the decision to go to war, is also critical when government
structures do not restrain such high-risk behavior on their own. Indeed, nondemocratic systems may not only fail to constrain
inappropriate aggressive behavior, they may even massively enable it by placing the resources of the state at the disposal of a
ruthless regime elite. It

is not that the first level of analysis, the individual, is unimportant - I have
already argued that it is important in elite perceptions about the permissibility and feasibility of
force and resultant necessary levels of deterrence. It is, instead, that the second level of analysis,
government structures, may be a powerful proxy for settings bringing to power those who are
disposed to aggressive military adventures and in creating incentive structures predisposed to
high-risk behavior. We might also want to keep open the possibility that a war/peace model focused on

democracy and deterrence might be further usefully refined by adding psychological profiles of
particular leaders as we assess the likelihood of aggression and levels of necessary deterrence .
Nondemocracies' leaders can have different perceptions of the necessity or usefulness of force and, as Marcus Aurelius should
remind us, not all absolute leaders are Caligulas or Neros. Further, the history of ancient Egypt reminds us that not all Pharaohs

the key to war


avoidance is understanding that major international war is critically an interaction, or synergy,
of certain characteristics at levels two and three - specifically an absence of [*395] democracy and
an absence of effective deterrence.
were disposed to make war on their neighbors. Despite the importance of individual leaders, however,

Yet another way to conceptualize the importance of democracy and deterrence in war avoidance is to note that each in its own way
internalizes the costs to decision elites of engaging in high-risk aggressive behavior. Democracy internalizes these costs in a variety
of ways including displeasure of the electorate at having war imposed upon it by its own government. And deterrence either
prevents achievement of the objective altogether or imposes

risk.

punishing costs making the gamble not worth the

n160

III. Testing the Hypothesis


Hypotheses, or paradigms, are useful if they reflect the real world better than previously held paradigms. In the complex world of
foreign affairs and the war puzzle, perfection is unlikely. No

general construct will fit all cases even in the


restricted category of "major interstate war;" there are simply too many variables. We should
insist, however, on testing against the real world and on results that suggest enhanced usefulness
over other constructs. In testing the hypothesis, we can test it for consistency with major wars .
That is, in looking, for example, at the principal interstate wars in the twentieth century, did
they present both a nondemocratic aggressor and an absence of effective deterrence? n161 And
although it, by itself, does not prove causation, we might also want to test the hypothesis against settings of potential wars that did
not occur. That is, in non-war settings, was there an absence of at least one element of the synergy? We might also ask questions
about the effect of changes on the international system in either element of the synergy. That is, what, in general, happens when a
totalitarian state makes a transition to stable democracy or vice versa? And what, in general, happens when levels of deterrence are
dramatically increased or decreased?

Accurate and verifiable scholarship shows low deterrence is the most important
correlate of war
Moore 4 Professor of Law at the University of Virginia. He formerly served as the first
Chairman of the Board of the United States Institute of Peace and as the Counselor on
International Law to the Department of State. (John Norton, Winter, Beyond the Democratic
Peace: Solving the War Puzzle, 44 Va. J. Int'l L. 341, Lexis Law)

there is strong evidence that deterrence - that is, the effect of external factors on the decision to go to war - is the
missing link in the war/peace equation. In my War & Peace Seminar, I have undertaken to examine the level
of deterrence before the principal wars of the twentieth century. My examination has led me to believe that in
every case the potential aggressor made a rational calculation that the war would be won , and won
As so broadly conceived,

n125

promptly. In fact, the longest period of time calculated for victory through conventional attack seems to be the roughly six weeks predicted by the German General Staff as the
time necessary to prevail on the Western Front in World War I under the Schlieffen Plan. Hitler believed in his attack on Poland that Britain and France would not take the
occasion to go to war with him. And he believed in his 1941 Operation Barbarossa against the Soviet Union that "[we] have only to kick in the door and the whole rotten structure
will come crashing down." n126 In contrast, following Hermann Goering's failure to obtain air superiority in the Battle of Britain, Hitler called off the invasion of Britain and
shifted strategy to the nighttime bombing of population centers, which became known as the Blitz, in a mistaken effort to compel Britain to sue for peace. Planners of the North

virtually all
principal wars in the twentieth century, at least those involving conventional invasion, were preceded by what I refer to as a
"double deterrence absence." That is, the potential aggressor believed that they had the military force
Korean attack on South Korea and Hussein's attack on Kuwait calculated that the operations would be complete in a matter of days. Indeed,

in place to prevail promptly and that nations that might have the military or diplomatic might to
prevent this were not inclined to intervene. This [*381] analysis has also shown that many of the perceptions we have about the origins of
n127

particular wars are flatly wrong. Anyone who seriously believes that World War I was begun by competing alliances drawing tighter should examine the real historical record of
British unwillingness to enter a clear military alliance with the French or to so inform the Kaiser. Indeed, this pre-World War I absence of effective alliance and resultant war
contrasts sharply with the later robust NATO alliance and an absence of World War III. n128

Considerable other evidence seems to support this historical analysis as to the importance of
deterrence. Of particular note, in 1995 Yale Professor Donald Kagan, a preeminent U.S. historian who has long taught a seminar on war, published a superb book On
the Origins of War and the Preservation of Peace.
In this book he conducts a detailed examination of the
Peloponnesian War, World War I, Hannibal's War, and World War II, among other case studies.
A careful reading of these studies suggests that each war could have been prevented by
achievable deterrence and that each occurred in the absence of such deterrence. Game theory seems to offer
n129

n130

yet further [*382] support for the proposition that appropriate deterrence can prevent war. For example, Robert Axelrod's famous 1980s experiment in an iterated prisoner's
dilemma, which is a reasonably close proxy for many conflict settings in international relations, repeatedly showed the effectiveness of a simple tit for tat strategy. n131 Such a
strategy is at core simply a basic method of deterrence, influencing behavior through incentives. Similarly, much of the game-theoretic work on crisis bargaining (and the danger
of asymmetric information) in relation to war and the democratic peace assumes the importance of deterrence through communication of incentives. n132 The well-known
correlation between war and territorial contiguity seems also to underscore the importance of deterrence and is likely principally a proxy for levels of perceived profit and
military achievability of aggression in many such settings.
It should further be noted that the democratic peace is not the only significant correlation with respect to war and peace, although it seems to be the most robust. Professors
Russett and Oneal, in recently exploring the other elements of the Kantian proposal for "Perpetual Peace," have also shown a strong and statistically significant correlation
between economically important bilateral trade between two nations and a reduction in the risk of war between them. Contrary to the arguments of "dependency theorists," such
economically important trade seems to reduce the risk of war regardless of the size relationship or asymmetry [*383] in the trade balance between the two states. In addition,
there is a statistically significant association between economic openness generally and reduction in the risk of war, although this association is not as strong as the effect of an
economically important bilateral trade relationship. n133 Russett and Oneal also show a modest independent correlation between reduction in the risk of war and higher levels of
common membership in international organizations. n134 And they show that a large imbalance of power between two states significantly lessens the risk of major war between
them. n135 All of these empirical findings about war also seem to directly reflect incentives. That is, a higher level of trade would, if foregone in war, impose higher costs in the
aggregate than without such trade, n136though we know that not all wars terminate trade. A large imbalance of power in a relationship rather obviously impacts deterrence and
incentives. Similarly, one might incur higher costs with high levels of common membership in international organizations through foregoing some of the heightened benefits of
such participation or otherwise being presented with different options through the actions or effects of such organizations.
These external deterrence elements may be yet another reason why democracies have a lower risk of war with one another. For their freer markets, trade, commerce, and
international engagement may place them in a position where their generally higher level of interaction means that aggression will incur substantial opportunity costs. Thus, the
"mechanism" of the democratic peace may be an aggregate of factors affecting incentives, both external as well as internal factors. Because of the underlying truth in the
relationship between higher levels of trade and lower levels of war, it is not surprising that theorists throughout human history, including Baron de Montesquieu in 1748,
Thomas Paine in 1792, John Stuart Mill in 1848, and, most recently, the founders of the European Community, have argued that increasing commerce and interactions among
nations would end war. Though by themselves these arguments have been overoptimistic, it may well be that some level of [*384] "globalization" may make the costs of war and
the gains of peace so high as to powerfully predispose to peace. Indeed, a 1989 book by John Mueller, Retreat From Doomsday, n137 postulates the obsolescence of major war
between developed nations (at least those nations within the "first and second worlds") as they become increasingly conscious of the rising costs of war and the rising gains of
peace.
In assessing levels of democracy, there are indexes readily available, for example, the Polity III n138 and Freedom House indexes. n139 I am unaware of any comparable index with
respect to levels of deterrence that might be used to test the importance of deterrence in war avoidance. n140 Absent such an accepted index, discussion about the importance of
deterrence is subject to the skeptical observation that one simply defines effective deterrence by whether a war did or did not occur. In order to begin to deal with the obvious
objections to this method and to encourage a more objective methodology for assessing deterrence, I encouraged a project to develop a rough but objective measure of
deterrence with a scale from -10 to +10 based on a large variety of contextual features that would be given some relative weighting in a complex deterrence equation before
applying the scaling to different war and non-war settings. n141 An innovative first effort uniformly showed high deterrence scores in settings where war did not, in fact, occur.
Deterring a Soviet first strike in the Cuban Missile Crisis produced a score of +8.5 and preventing a Soviet attack against NATO produced a score of +6. War settings, however,
produced scores ranging from -2.29 (Saddam Hussein's decision to invade Kuwait in the Gulf War), -2.18 (North Korea's decision to invade South Korea in the Korean War),
-1.85 (Hitler's decision to invade Poland in World War II), -1.54 (North Vietnam's decision to invade South Vietnam following the Paris Accords), -0.65 (Slobodan Milosevic's
decision to defy NATO in Kosovo), +0.5 (the Japanese decision to attack Pearl Harbor), +1.25 (the Austrian decision, encouraged by Germany, to attack Serbia,
which [*385] was the real beginning of World War I), to +1.75 (the German decision to invade Belgium and France in World War I). As a further effort at scaling and as a point
of comparison, I undertook to simply provide an impressionistic rating based on my study of each pre-crisis setting. That produced high positive scores of +9 for both deterring a
Soviet first strike during the Cuban Missile Crisis and NATO's deterrence of a Warsaw Pact attack and even lower scores than the more objective effort in settings where wars
had occurred. Thus, I scored North Vietnam's decision to invade South Vietnam following the Paris Accords and the German decision to invade Poland at the beginning of World
War II as -6. I scored the North Korean/Stalin decision to invade South Korea in the Korean War as -5, the Iraqi decision to invade Kuwait as -4, Milosevic's decision to defy
NATO in Kosovo and the German decision to invade Belgium and France in World War I as -2, and the Austrian decision to attack Serbia and the Japanese decision to attack
Pearl Harbor as -1. Certainly even knowledgeable experts would be likely to differ in their impressionistic scores on such pre-crisis settings, and a more objective methodology
for scoring deterrence would be valuable. Nevertheless, both exercises did seem to suggest that deterrence matters and that high levels of deterrence can prevent war.
Yet another piece of the puzzle, which may clarify the extent of deterrence necessary in certain settings, may also assist in building a broader hypothesis about war. In fact, it has
been incorporated into the efforts at scoring deterrence just discussed. That is, newer studies of human behavior are increasingly showing that certain perceptions of decision
makers can influence the level of risk they may be willing to undertake. It now seems likely that a number of such insights about human behavior in decision making may be
useful in considering and fashioning deterrence strategies. Perhaps of greatest relevance is the insight of "prospect theory," which posits that individuals evaluate outcomes with
respect to deviations from a reference point and that they may be more risk-averse in settings posing potential gain than in settings posing potential loss. n142 The evidence of this
"cognitive bias," [*386] whether in gambling, trading, or, as is increasingly being argued, foreign policy decisions generally, is significant. Because of the newness of efforts to
apply a laboratory-based "prospect theory" to the complex foreign policy process generally, and particularly due to ambiguities and uncertainties in framing such complex
events, our consideration of it in the war/peace process should certainly be cautious. It does, however, seem to elucidate some of the case studies.
In the war/peace setting, "prospect theory" suggests that deterrence may not need to be as strong to prevent aggressive action leading to perceived gain. For example, there is
credible evidence that even an informal warning to Kaiser Wilhelm II from British Foreign Secretary Sir Edward Grey, if it had come early in the crisis before events had moved
too far, might have averted World War I. And even a modicum of deterrence in Kuwait, as was provided by a small British contingent when Kuwait was earlier threatened by an
irredentist Iraqi government in 1961, might have been sufficient to deter Saddam Hussein from his 1990 attack. Similarly, even a clear U.S. pledge to defend South Korea before
the attack might have prevented the Korean War. Conversely, following the July 28 Austrian mobilization and declaration of war against Serbia in World War I, the issue for

Austria may have begun to be perceived as loss avoidance, thus requiring much higher levels of deterrence to avoid the resulting war. Similarly, the Rambouillet Agreement may
have been perceived by Milosevic as risking loss of Kosovo and his continued rule of Serbia, and, as a result, may have required higher levels of NATO deterrence to prevent
Milosevic's actions in defiance. Certainly NATO's previous hesitant response against Milosevic in the Bosnia phase of the Yugoslav crisis did not create a high level of
deterrence. n143 One can only surmise whether the [*387] killing in Kosovo could have been avoided had NATO taken a different tack, both structuring the issue less as loss
avoidance for Milosevic and considerably enhancing deterrence. Suppose, for example, NATO had emphasized that it had no interest in intervening in Serbia's civil conflict with
the Kosovo Liberation Army (KLA) but that it would emphatically take action to punish massive "ethnic cleansing" and other humanitarian outrages, as had been practiced in
Bosnia. And on the deterrence side, suppose it made clear in advance that any NATO bombardment would be severe, that ground troops would be introduced if necessary, that
in any assault it would pursue a "Leadership Strategy" focused on targets of importance to Milosevic and his principal henchmen (including their hold on power), and that unlike
earlier in Bosnia, NATO immediately would seek to generate war crime indictments of all top Serbian leaders implicated in any atrocities. The point here is not to second-guess
NATO's actions in Kosovo but to suggest that taking into account potential "cognitive bias," such as "prospect theory," may be useful in fashioning effective deterrence. "Prospect
theory" may also be relevant in predicting that it is easier to deter (that is, lower levels are necessary) an aggression than to undo that aggression. Thus, much higher levels of
deterrence were probably required to compel Saddam Hussein to leave Kuwait than to prevent him from initially invading that State. In fact, not even the presence of a powerful
Desert Storm military force and a Security Council Resolution directing him to leave caused Hussein to voluntarily withdraw. As this [*388] real world example illustrates,
there is considerable experimental evidence in "prospect theory" of an almost instant renormalization of reference point after a gain. That is, relatively quickly after Saddam
Hussein took Kuwait, a withdrawal was framed as a loss setting, which he would take high risks to avoid. Indeed, we tend to think of such settings as settings of compellance,
requiring higher levels of incentive to achieve compulsion producing an action, as opposed to lower levels of deterrence needed for prevention.
One should also be careful not to overstate the effect of "prospect theory" or fail to assess a threat in its complete context. We should remember that a belated pledge by Great
Britain to defend Poland before the Nazi attack did not deter Hitler, who believed under the circumstances that the British pledge would not be honored. It is also possible that
the greater relative wealth of democracies, which have less to gain in all out war, is yet another internal factor contributing to the "democratic peace." n144 In turn, this also
supports the extraordinary tenacity and general record of success of democracies fighting in defensive settings, as they may also have more to lose.
In assessing the adequacy of deterrence to prevent war, we might also want to consider whether extreme ideology, strongly at odds with reality, is a factor requiring higher levels
of deterrence for effectiveness. One example may be the extreme ideology of Pol Pot, which led to his false belief that his Khmer Rouge forces could defeat the Vietnamese. n145 He
apparently acted on that belief in a series of border incursions against Vietnam that ultimately ended in his losing a war. Similarly, Osama bin Laden's 9/11 attack against
America, hopelessly at odds with the reality of his defeating the Western World and producing for him a strategic disaster, seems to have been prompted by his extreme ideology
rooted in a distorted concept of Islam at war with the enlightenment. The continuing suicide bombings against Israel, encouraged by radical rejectionists and leading to fewer
and fewer gains for the Palestinians, may be another example. If extreme ideology is a factor to be considered in assessing levels of deterrence, it does not mean that deterrence
is doomed to fail in such settings but only that it must be at higher levels (and properly targeted toward the relevant [*389] decision elites behind the specific attacks) to be
effective, as is also true in perceived loss or compellance settings. n146
Even if major war in the modern world is predominantly a result of aggression by nondemocratic regimes, it does not mean that all nondemocracies pose a risk of war all, or
even some, of the time. Salazar's Portugal did not commit aggression. Nor, today, do Singapore or Bahrain or countless other nondemocracies pose a threat. That is, today

nondemocracy comes close to a necessary condition in generating the high-risk behavior leading
to major interstate war. But it is, by itself, not a sufficient condition for war. The many reasons for this, of course,
include a plethora of internal factors, such as differences in leadership perspectives and values, size of military, and relative degree of the rule of law, as well as levels of external

where an aggressive nondemocratic regime is present and poses a credible military


threat, then it is the totality of external factors, that is, deterrence, that become crucial .
deterrence. n147 But

2AC PAN K
No impact to China threat discourse bad reps dont influence policies towards
China and our advantage solves the impact
Wenxin 5Zhang Jiye and Chen Wenxin, Xiandai Guoji Guanxi, Ocnus, September 20, 2005,
p. http://www.ocnus.net/cgibin/exec/view.cgi?archive=78&num=20415
Nevertheless, no matter the extent of playing up the "China military threat theory" by the US
military, its influence on Sino-US relations and the development of the international situation is
still limited. From the 1990s to now, the "China threat theory" had emerged once every two to
three years. The US rightist forces and the US military are accustomed to using it as a "target" to
play up the "China military threat theory" in order to consolidate their sphere of influence and
position in the US political arena. However, the main trend of the development of Sino-US
relations has not been seriously influenced. On the one hand, in the US political field, besides
military intelligence and rightist groups that publicize the "China military threat theory," there
are many officers and scholars who "calmly view the situation across the ocean," seriously look at China's
development, and call for strengthening contacts between the United States and China. A noted US think tank, the Rand Corporation, on 19 May
submitted an evaluation report to the US Air Force on "China's Defense Modernization: Opportunities and Challenges." The report holds that the
Pentagon's evaluation of China's military spending is seriously "inflated" and the practice of playing up "China military threat theory" on purpose
should be rectified. On 26 May, US congressional heavyweight Senator Joe Lieberman of the Democratic Party and Republican congressman Alexander
jointly put forward an "Act on Cultural Exchange Between the United States and China in 2005" and asked the US Government to appropriate $1.3
billion from FY2006 to FY2011 for the promotion of cultural exchanges between the United States and China, especially for the expansion and
strengthening of US education in Chinese language and a program for exchange students between the two countries. In order to guarantee the
implementation of the exchange plan, the proposal also suggested establishing the United States-China Engagement Strategy Council. In introducing
the act, Senator Lieberman said: "All

misunderstanding between China and the United States can be solved


by engagement between the two countries." Even within the US Government, views toward the "China military threat theory"
are different. The "engagement group," headed by the Department of State and the National Security Council, holds a different view on the "China
military threat theory," and so the US Department of Defense could not but postpone its publication of the "annual report on Chinese military power"
again and again. On the other hand, due to pragmatic political considerations, the US Government needs to consider United States-China relations
based on the overall interests of the country. Although the United States has made some achievements in its global war against terrorism in the current
phase, the new round of terrorist attacks in Britain shows the United States still cannot extricate itself from the war against terrorism and will need
China's support.

Our knowledge of China is accuratetheir authors have flawed information


Chan 4PhD in Political Science from Minnesota U, Professor and Chair of the Department of Political Science at Colorado U at
Boulder (Steve, Asian Affairs, Vol 31, No. 3 (Fall, 2004), Extended Deterrence in the Taiwan Strait: Learning from Rationalist
Explanations in International Relations, JSTOR, http://www.jstor.org/stable/30172621, p. 167, RBatra)

Rationalist interpretations do not imply that people are omnipotent in their ability to procure and process information. We know all
too well that people are subject to a variety of cognitive and perceptual errors (for example, Jervis 1976; Levy 1997; Kahneman and
Tversky 2000; Tversky and Kahneman 1977). This recognition of limits to rationality, however, hardly warrants general attributions

officials
know their counterparts far better than scholars may wish to acknowledge. Washington, Beijing,
and Taipei, for instance, invest enormous time, effort, and resources in trying to gain an accurate
understanding of each other. Academics have a hard time claiming any special insight or
unique source of wisdom, whether it is based on mastery of the other side's language, intimate
familiarity with its culture, or access to timely and sensitive information with restricted
distribution. If anything, they are usually at a considerable disadvantage on these scores when
compared to diplomats, intelligence analysts, and even journalists and business people. Indeed,
academics in fields such as history and political science typically operate in the realm of
of naivet , even stupidity, to government leaders. On the contrary, it seems sensible to start from the premise that

common knowledge, outdated information, and mundane data . This confession in turn implies that at least
for some of us, our individual and collective forte lies with the analysis of persistent empirical patterns and the formulation of
general models of foreign policy conduct.

2AC SOFT POWER


Soft power in the context of China is a useful heuristic for understanding and
working within the political
William Callahan 15 - London School of Economics and Political Science, (Identity and
Security in China: The Negative Soft Power of the China Dream, 2015 VOL 35(3-4) 216229)
hk
Certainly, much of the discussion of Chinas soft power highlights the positive attractive nature
of Chinese culture, values and foreign policy. What is under-researched is the theoretical dynamics
of positive/negative and domestic/foreign. Rather than take for granted that we understand
what the Chinese values are that inform the PRCs soft power, debates over the sources of soft
power show how Chinese values are being actively produced in an international dialogical
process. Hence soft power is about more than the export of pre-existing essential values; it also
involves the production of values both at home and abroad. Like in America, Chinas soft power
actually takes shape through the romanticisation of a particular national culture into universally
desirable values. Before it can spread values abroad, soft power policy first needs to produce
and police values at home. Soft power thus is not an entity that can be empirically measured, so
much as a domestic process of social construction that defines the symbolic borders of self and
Other, and thus of identity and security (see Callahan, 2010; Campbell, 1998; Connolly, 1991,
pp. 3663; Walker, 1993). Following this line of argument for the Japanese case, David Leheny
(2006, p. 223) feels that the concept of soft power has less value as a tool for evaluating Japans
regional importance than it does as a heuristic device for grasping how Japanese policymakers
now see their regional role. I would push this argument one step further to suggest that soft
power discourse is a useful heuristic device for understanding how Chinese policy makers
and public intellectuals are actively constructing a China and a world to promote their
ideological projects. In other words, soft power is primarily an issue of domestic politics
determining Chinas future direction and only secondarily about international politics. While
Chinese discussions of soft power certainly seek to build favour among foreign audiences, they
are also concerned with the identity/security issue of safeguarding regime legitimacy at home
(see Edney, 2015). This domestic focus is part of what Chinese Foreign Affairs University Vice
President Qin Yaqing calls the PRCs identity dilemma. Rather than worrying about how the
PRC fits into international institutions like the World Trade Organisation or the United Nations,
Qin (2006) argues that the main issue for the PRCs engagement with the world is the identity
politics of answering the question Who is China?. This identity dilemma is part of a broad and
ongoing debate in the PRC about the moral vacuum that China faces after three decades of
economic reform and opening up. In other words, intellectuals from across the political
spectrum liberals, socialists, traditionalists and militarists all worry about the values crisis
presented by what they call Chinas new money-worship society (see Liu, 2010; Pan, 2009; Xu,
2011). The heart of Chinese foreign policy thus is not a security dilemma between great powers,
but an identity dilemma within China as people ask Who is China? and What kind of world
does it want? (Qin, 2006, p. 13). The domestic focus of soft power and foreign policy also leads
us to the other contingent dynamic: positive/negative. Certainly, Chinese culture is presented as

a positive treasure box of soft power resources, and Chinese history as a positive example of
peace and development that is still relevant today. Chinas domestic policy of harmonious
society and its foreign policy of peacefully rising in a harmonious world are all based on the
idealised view of Chinese civilisation as open to the world and tolerant of outsiders. Peaceful
rise refers not just to recent experience, but looks to Chinas imperial history as the benevolent
great power that presided over hundreds of years of peace in East Asia (see Callahan, 2012;
Kang, 2007; Katzenstein, 2012; Rozman, 2013). Chinese soft power discourse thus looks to
traditional Chinese civilisation as a resource for the countrys values in the twenty-first century ,
especially the values of peace and harmony Yet alongside this positive view of a benevolent
China that embraces the outside world, identity and security are linked in the negative process
of drawing symbolic borders between self and Other. Rather than a set of stable essential
values, civilisation here is better understood as a contingent discourse that takes shape in
relation to its opposite: barbarism. As political theorist Walter Benjamin (1968, pp. 256257)
argues: There is no document of civilization which is not at the same time a document of barbarism. In these contingent self/Other relations, whenever we declare something to
be civilised, we are simultaneously declaring something else barbaric (Campbell, 1998; Todorov, 2010). While Chinese texts often talk about 5,000 years of civilisation, it is
necessary to recognise that in order to affirm civilisation, they first need to create and then exclude barbarians.

Having complex views of soft power is uniquely good to talk about the contingent
relations of engagement
William Callahan 15 - London School of Economics and Political Science, (Identity and
Security in China: The Negative Soft Power of the China Dream, 2015 VOL 35(3-4) 216229)
hk
This article has argued that we need to have a more complex view of soft power . First it
questioned the dominant view that soft power is an entity, a variable, which can be empirically
measured. It argued that soft power is best understood as a social construction that can tell us
about identity and security dynamics, in particular the contingent relations of hard/soft power,
positive/negative strategies, and foreign/domestic politics. While mainstream soft power theory
looks to idealised notions of the self (e.g. national image) that are exported to benefit foreign
relations, the article used Chinese texts to develop the idea of negative soft power, where
identity is constructed by excluding difference in an identity/security dynamic that primarily
works in domestic space. Chinese civilisation characteristically is seen as a major source of the
PRCs soft power; it is argued here that civilisation is not an entity, but generally takes shape
against the negative idealisation of barbarism. Even very positive discourses like the China
dream rely on mobilising negative images of the foreign as barbaric. Although Chinese soft
power discourse generally works in domestic space to generate national identity and regime
legitimacy, Chinas negative soft power strategy is increasingly going global due to a
combination of factors especially Chinas new wealth and confidence in the context of
economic and political crises in Europe and the US since 2008. One of the most prominent
aspects of Chinas soft power policy is the spread of Confucius Institutes around the world since
2004. According to Vice-Minister Xu Lin, the DirectorGeneral of the Confucius Institute
Headquarters (CIH), Confucius Institutes are the brightest brand of Chinas soft power (2006:
Kongzi xueyuan, 2007). But as events at the European Association of Chinese Studies (EACS)
biennial conference in 2014 showed, even Chinas brightest brand employs negative soft power
strategies. CIH was one of the co-sponsors of the conference, where Xu gave a keynote speech.

According to a report by the EACS president, Xu was upset by some of the paper topics and
dismayed by the prominent display in the conference materials of information about Taiwanese
sponsors. Xus solution was to steal all of the programmes and tear out pages that referred to
CIH, Taiwans Chiang Ching-Kuo Foundation and the Taiwan National Central Library. When
EACS President Roger Greatrex discovered this censorship, he ordered that copies of the excised
pages be distributed to all conference participants. The EACS report concluded by proclaiming
that Censorship of conference materials cannot and will never be tolerated by the EACS
(Greatrex, 2014a; 2014b). Such censorship was seen as a setback for Chinas soft power in many
journalistic commentaries (Beijings Propaganda, 2014; Cai, 2014; Redden, 2014). But back in
Beijing, Vice Minister Xu was cheered on by the Global Times, which saw her page-tearing as a
heroic patriotic action in the fight against Taiwanese independence (Hanban zhuren, 2014).
Once again, the main audience for soft power activities, even those of Chinas brightest brand, is
not outside China, but inside the PRC. It works through the negative strategy of censoring
academic materials, rather than the positive strategy of spreading Chinese civilisation. Trouble
in Europe does not matter as much as success in Beijing. The China dream likewise informs soft
power discourse that is very popular within the PRC (Cai, 2013b; Ma Haiyan, 2013), but which
gains little traction abroad. This makes sense as it promotes largely negative portrayals of
foreign countries in order to mobilise Chinas domestic audience (see Zhang, 2014). In this way,
the China dreams negative soft power evokes a form of nationalism that is employed to
safeguard the CCPs regime legitimacy (see Edney, 2015). China Dream discourse thus combines
the insights of Edneys and Yablokovs (2015) articles: soft power is generated through the
negative dynamic of conspiracy theories in the service of building national cohesion in
domestic space. While the PRC is strong in economic and military terms, its regime security as
fragile superpower is more tenuous (Shirk, 2008). Hence soft power in China takes on more
negative forms that are directed at a domestic audience. Yet according to Nyes version of soft
power, foreign audiences are crucial; if soft power products are not attractive to them, then the
soft power strategy is unsuccessful. Certainly, we could follow the current academic trend to
celebrate how China has adopted and adapted the soft power concept to suit its needs. But if a
goal is to turn enemies into friends, then it is not working very well. Here the PRC is a partial
power whose global influence is broad, but thin (Shambaugh, 2013, p. 268). This is a major
problem for soft power in hard states.

2AC WARMING DISCOURSE GOOD


Increased engagement in public discourse over climate change influences the way
we think about climate change
Brian Shreck & Arnold Vedlitz 16 Brian is a part of the National Wind Institute &
Department of Political Science, Texas Tech University, Lubbock, Texas, USA and Arnold is part
of the Bush School of Government and Public Service, Institute for Science, Technology and
Public Policy, Texas A&M University, College Station, Texas, USA (The Public and Its Climate:
Exploring the Relationship Between Public Discourse and Opinion on Global Warming, 11 July
2016, 29:5, 509-524, DOI: 10.1080/08941920.2015.1095380) hk
For the past 25 years, democratic theorists have been advocating models of democracy in which public deliberation and public
discourse are the cornerstones of democratic legitimacy. To this end, deliberative

democrats argue that citizens


ought to participate in a meaningful way in the deliberation and discussion of proposed laws and
policies. Embedded in these normative theories is a set of empirical assumptions and
prerequisite conditions. This study has presented an empirical examination of some of the observable implications of
deliberative theory. As we have shown here, the answers to these questions carry direct implications for how democratic
societies solve scientifically complex, politically contested policy issues, which include not just
climate change, but also other environmental, science, and technology issues. We have found that while
high levels of engagement in public discourse are positively associated with holding a strong
belief in the reality of anthropogenic climate change, they are not associated with strong beliefs on the denial end
of the spectrum. Meanwhile, while objectively measured knowledge is negatively associated with strong denial of the reality of
climate change, self-assessed knowledge is strongly positively associated with strong beliefs on both sides of the issue. However, with
engagement in public discourse, we see a slightly different pattern when we look at preferences for climate change-related policies.
Here, engagement

in public discourse is associated with strong opinions both in support and in


opposition. One possible conclusion that we might draw from this is that increased engagement in public
discourse might be associated with some emerging consensus on the reality of
anthropogenic climate change, but not on the policy debates regarding what should be done about it. However,
we find a non-statistically significant pattern in the data that suggests public discourse might help mitigate the
strong influence of partisanship in determining policy opinions related to climate change. These
results should not be generalized beyond the present sample because they do not reach statistical significance. However, our data
show a pattern in which both Democrats and Republicans who have engaged in more public discourse than their fellow partisans
seem more likely to hold policy opinions about climate change that are not aligned with their party identifications. If this
relationship holds true in the population, it would suggest that through discourse

individuals might come into


contact with new information and arguments and that, at least for a few, these experiences can
lead them to reflect critically on their previously held beliefs and opinions . This would be
consistent with the idea that increased public discourse will increase the diversity of opinions
and perspectives that are represented in the debate. This adds context to the findings that, overall, suggest an
echo chamber effect by which increased public discourse on climate change might be associated with a
softening of the strong effects of partisanship. However, this softening effect does not seem to be strong enough to
break through the intense polarization on the issue. This conclusion comes with the caveat that further study is needed to determine
whether this pattern appears consistently in other studies, or whether it is merely an artifact of this particular sample.

Framing in the context of environmental movements key to actualize change


George Lakoff 10 - professor of linguistics at the University of California, Berkeley (Why it
Matters How We Frame the Environment, 17 Mar 2010, Environmental Communication, 4:1,
70-81, DOI: 10.1080/17524030903529749) hk
Framing for an Environmental Movement Successful social movements require the coherence
provided by coherent framing. Think of the union movement, the anti-war (or peace) movement,
the civil rights movement, or the feminist movement. The basic ideas are simple and
straightforward. Unions: Because companies have much more power over individual workers in
matters of pay, benefits, and working conditions, workers need to join together in unions to
equalize that power. Civil rights: African-Americans have been denied a wide range of civil
rights and should have them. Feminism: Women have been relegated to inferior positions in
society and deserve equality in those areas. The same should be true of environmentalism.
Environmentalism: The natural world is being destroyed and it is a moral imperative to preserve
and reconstitute as much of it as possible as soon as possible. Of course, in each case,
complexities have arisen, along with powerful reactions. The details are enormously complex in
each, as they are in environmentalism. But what has made social movements effective is a simple
basic framing. The social movement approach is idealistic of necessity . Idealism mobilizes.
And it throws a light on, and presents a counterweight to, moral compromise. The media reports
mainly on political compromise, as exemplified by the KerryGraham quote we started with.
Without a clearly framed social movement, the moral compromise behind the political
compromise can be hidden. Truth must be framed effectively to be seen at all. That is why an
understanding of framing matters.

1AR WARMING DISCOURSE GOOD


Securitization is important to dialogue and debate over climate change in China
Jonna Nyman & Jinghan Zeng 16 Jonna: Department of Politics and International
Relations, University of
Leicester and Jinghan: Department of Politics and International Relations, Royal Holloway,
University of London (Securitization in Chinese climate and energy politics, 24 JAN 2016
online, WIREs Clim Change, 7: 301313. Wiley online library) hk
Overall,

the literature on the domestic aspects of climate change is more positive toward
securitizing the issue or viewing it as a threat to security.65 Zhang argues in favor of China adopting a broader
notion of national security to include climate change.65 Wang Tao argues that China should use a
development perspective to coordinate energy security and climate security as
they overlap. 48 He suggests energy security involves providing sustainable, reliable energy, which
needs to take future climate change into consideration , while the goal of climate security is to manage climate
risks and thus achieve security and sustainable development of human society, which contains the key elements of energy security.
Along a similar vein, Liu Yin argues that global

warming poses problems for and pressures on Chinas


energy security, but can also bring opportunities to force China toward a more sustainable energy system.66 There is also focus
on climate change as a threat to Chinas food security, and so by extension, to security more broadly. A focus on climate change as a
nontraditional security issue is a common theme in the domestic debate. Wang analyses how ecological issues, including climate
change, are securitized.67 She notes that nontraditional issues like climate change are increasingly upgraded in importance and
considered issues of security in China. Na Li and Yang Nan argue that

climate change is a threat to both

traditional and nontraditional security ,68 climate change will intensify territorial disputes
among countries and also threatens nontraditional security including food security, water
shortage, human health, and climate migrants. Wang et al. argue that climate change is closely related
with national security, and that China in fact uses a new concept of security which includes paying
attention to increasing interaction between climate change, energy security, food, and water security, as well as other emerging
security concerns.63

Security and securitization are increasingly important in debates

over climate change in China, but a wide range of perspectives is forwarded. Alongside the split between literature
focusing on the international level and those concerned with climate change at the national or domestic level, there is increasing
focus on a wider notion of security or nontraditional security. While

the focus is often on national security,


sometimes the threat emphasized is to Chinas economy, energy security or resources, with food
and water raised as central concerns, all of which are also at times seen as part of national
security.

Prioritizing climate strategies is good the U.S. is in a unique position of power to


influence other countries in a pragmatic way
Joshua Busby 16 - Associate Professor of Public Affairs, University of Texas at Austin
(Sustainable Security: Rethinking American National Security Strategy, 2016,
http://tobinproject.org/sites/tobinproject.org/files/assets/Sustainable%20Security%20-%20All
%20Chapters.pdf) hk

The United States,

as the most influential state in the international system, can at least rally the
resources to begin the process of better understanding the vulnerabilities and helping countries jump-start the
planning process.141 In June 2015, the Obama administration announced a new thirty-four million dollar initiative with
partner organizations that included inkind contributions from the Red Cross, Google, and ESRI to support climate contingency
measures in three vulnerable developing countries, Bangladesh, Colombia, and Ethiopia.142 Such investments in the millions can
minimize if not eliminate the number of multi-million and/or billion dollar emergency disaster responses later, conserving US
strategic assets and financial resources for other purposes. All

of this is happening at a potentially dangerous


moment in the international system as the relative economic advantage of the United States
declines as China rises. As the United States becomes more selective in its choice of areas of
strategic investment, climate change has to be one of those areas.
States have strong incentives to cooperate over climate change specifically true
in the context of China
Joshua Busby 16 - Associate Professor of Public Affairs, University of Texas at Austin
(Sustainable Security: Rethinking American National Security Strategy, 2016,
http://tobinproject.org/sites/tobinproject.org/files/assets/Sustainable%20Security%20-%20All
%20Chapters.pdf) hk
The United States and China are going to be the primary recipients of that fingerpointing. Climate change
presents a curious problem for the two countries, especially given the power transition process that is currently underway with the United States
experiencing relative decline and China a rapid ascent. We know from history that such moments are especially dangerous times when fading
hegemons face revisionist challengers seeking to re-make the world in their image. Yet, that logic of relative gains and preoccupation with narrow
national selfinterest runs against the wider liberal logic of mutual gains from trade.118 Moreover, climate

protection is a global
public good; neither the US nor China, despite being the largest emitters, can resolve the
problem on their own. From the standpoint of selfprotection against global climate change ,
states have strong incentives to band with coalitions of the relevant (i.e. the major emitters) to stave
off suboptimal outcomes for everyone. Despite its relative decline, the United States is, as Bruce Jones notes, the largest minority
shareholder in the international order, and, as such, still possesses sufficient capability to lead by example and shepherd consolidation of a the liberal

even as there is inevitable friction over


Chinas ambitions in the South China Sea, the US and China possess shared goals and
vulnerabilities because of economic interdependence and exposure to climate hazards . In this
context , the countries need to rise above their sources of discord and enter into a
international order in which China is embedded.119 As a consequence,

mature relationship of quasi-rivalry on some dimensions of policy and cooperation on others. Even in the climate space, the United
States may have to engage in technical cooperation on emissions monitoring, but at the same time prepare for differential ambition on climate change
by exploring coercive instruments such as border tax adjustments to induce greater Chinese engagement.120

You might also like